nurs 200 final prepu questions

Réussis tes devoirs et examens dès maintenant avec Quizwiz!

At a class for new mothers, the nurse notes that the participants need more teaching when one of the mothers makes which comment? a. "I have to stop breastfeeding because of mastitis." b. "I can use warm compresses to ease the discomfort." c. "Breastfeeding every four hours appears to help." d. "I have been using a breast pump after feeding my baby."

a. "I have to stop breastfeeding because of mastitis." The nurse should teach the women about the etiology of mastitis and encourage breast-feeding, emphasizing that it is safe for infants. Continued emptying of the breast or pumping improves the outcome, decreases the duration of symptoms, and decreases the incidence of breast abscess.

A client is to have a vaginal hysterectomy to repair her stage IV uterine prolapse. The nurse realizes she needs more education when she states: a. "I'm not going home with a Foley catheter in place." b. "The leg bag will be covered by my clothes, once I am home." c. "My husband is not crazy about the six weeks of pelvic rest." d. "I have already picked up the stool softeners."

a. "I'm not going home with a Foley catheter in place." A Foley catheter will be left in place for up to one week to allow the surgical site time to heal properly. The client may also experience some dysuria once it is removed due to the swelling that can occur. The client will need instructions on how to properly care for a leg bag, the importance of pelvic rest to allow for proper healing, and using stool softeners to avoid straining or stressing the pelvic region.

A nurse is reviewing the various treatment options with a client diagnosed with uterine fibroids. The nurse determines that the teaching was successful based on which statement? a. "If I use hormone therapy, my fibroids may grow back when I stop the medication." b. "A myomectomy will not allow me to keep my uterus." c. "Uterine artery embolization is associated with minimal pain." d. "Laser surgery won't affect my ability to have children."

a. "If I use hormone therapy, my fibroids may grow back when I stop the medication." Typically, with hormonal therapy, fibroids regrow when the medication is stopped. A myomectomy preserves the uterus. Uterine artery embolization is frequently painful. Laser surgery can cause scarring and adhesions which could impact future fertility.

A woman with an artificial mitral valve develops heart failure at the 20th week of pregnancy. Which measure would the nurse stress with her during the remainder of the pregnancy? a. obtaining enough rest b. maintaining a high fluid intake c. beginning a low-impact aerobics program d. discontinuing her prepregnancy anticoagulant

a. obtaining enough rest As the blood volume doubles during pregnancy, heart failure can occur. The pregnant woman needs to obtain adequate rest to prevent overworking the heart. Fluid may need to be restricted.

A client diagnosed with pelvic organ prolapse is being taught how to perform pelvic floor muscle exercises. During the teaching session, the client asks the nurse, "How do these exercises help?" Which response by the nurse would be most appropriate? a. "They help to increase the volume of your muscles which leads to stronger muscle contraction." b. "They help to move the pelvic floor upward so that your symptoms eventually decrease." c. "The exercises increase the amount of blood that your muscles receive making them less relaxed." d. "The exercises help you to establish regular bowel elimination patterns so you don't strain so much."

a. "They help to increase the volume of your muscles which leads to stronger muscle contraction." The purpose of pelvic floor exercises is to increase the muscle volume, which will result in a stronger muscular contraction. The exercises do not move the pelvic floor upward, increase blood supply, or establish regular elimination patterns.

How should the nurse counsel a postpartum client on how to prevent mastitis? a. "Wash your hands thoroughly, and let your breasts dry after each feeding." b. "If you notice that your breast is warm, hard, or red, stop feeding on that side and pump from that breast instead." c. "Be sure to keep your breasts covered when you are not feeding or pumping." d. "Sterilize your bottles and pump equipment after each use."

a. "Wash your hands thoroughly, and let your breasts dry after each feeding." Handwashing is one of the best ways to prevent infection. If the woman feels that her breast is warm, hard, or red, she should increase the amount of breastfeeding from that side. It is not necessary to sterilize bottles and pumping equipment after each use. Normal dish washing is sufficient. Keeping the breasts exposed to the air to dry will aid in preventing infection.

A 25-year-old woman is at the primary care provider for her annual check up. The nurse educated the woman on risks for cervical cancer. Which question would be important to ask as part of a risk screening? a. "Were you sexually active at an early age?" b. "Do you have a history of high blood pressure?" c. "Have you had problems trying to get pregnant?" d. "How long have you been severely overweight?"

a. "Were you sexually active at an early age?" Women that have a history of sexual activity within the first year of getting their menstrual cycle are at increased risk for cervical cancer later in life. Infertility, obesity, and high blood pressure put women more at risk for endometrial cancer.

A client with asthma is confused by her primary care provider continuing her medication while she is pregnant, since she read online the medications can cause birth defects. What is the nurse's best response? a. "Your primary care provider will order safe doses of your medication." b. "It's OK to not use them if you would feel more comfortable." c. "They won't cause any major defects." d. "I'll let your primary care provider know how you feel about it."

a. "Your primary care provider will order safe doses of your medication." Women should take no medication during pregnancy except that prescribed by their primary care provider. The PCP will work with the mother to ensure the safest amount is given to adequately handle the mother's health issues and not injure the fetus. The PCP must weigh the risks against the benefits for both the mother and her fetus. The nurse should not encourage the client to stop her asthma medication as that may result in the client having an asthma attack, which could result in injury to the fetus or even miscarriage. The nurse should not tell the client a drug will not cause any defects, especially if it is known that it can. That could make the nurse liable for damages. The nurse should inform the PCP of the client's concerns; however, it is more important for the nurse to calm the client's anxiety and offer positive reinforcement that the PCP is working hard to protect the mother and infant from harm.

The client arrives in the office and reports that her feet and legs swelling. During a client evaluation, the nurse notes that she can elicit a 4-mm skin depression that disappears in 10 to 15 seconds. The nurse correctly documents this finding as: a. 2+ pitting edema. b. 1+ pitting edema. c. 3+ pitting edema. d. 4+ pitting edema.

a. 2+ pitting edema. Pitting edema is recorded using the following relative scale: 1+ is a 2-mm depression that disappears rapidly; 2+ is a 4-mm depression that disappears in 10 to 15 seconds; 3+ is a 6-mm depression that lasts more than one minute; and 4+ is an 8-mm depression that lasts 2 to 3 minutes.

A nurse is assessing a newborn's temperature. Which reading would the nurse document as normal? a. 37.0° C (98.6° F) b. 36.0° C (96.8° F) c. 35.0° C (95.0° F) d. 38.0° C (100.4° F)

a. 37.0° C (98.6° F) On average a newborn's temperature ranges from 36.5° C to 37.5° C (97.9° F to 99.7° F).

A client has undergone an abdominal hysterectomy to remove uterine fibroids. Which interventions should a nurse perform as a part of the postoperative care for the client? Select all that apply. a. Administer analgesics promptly and use a patient-controlled analgesia (PCA) pump. b. Avoid pillows and changing positions frequently. c. Avoid intake of excess carbonated beverages in the diet. d. Ambulate frequently. e., Administer antiemetics to control nausea and vomiting.

a. Administer analgesics promptly and use a patient-controlled analgesia (PCA) pump. d. Ambulate frequently. e., Administer antiemetics to control nausea and vomiting. The postoperative care plan for a client who has undergone a hysterectomy includes administering analgesics promptly and using a PCA pump, frequent ambulation, and administering antiemetics to control nausea and vomiting. The nurse should change the position of the client frequently and use pillows for support to promote comfort and pain management. Ambulation is key in the prevention of postoperative complications. An excess of carbonated beverages in the diet does not affect the postoperative healing process.

A nurse in the maternity triage unit is caring for a client with a suspected ectopic pregnancy. Which nursing intervention should the nurse perform first? a. Assess the client's vital signs. b. Administer oxygen to the client. c. Obtain a surgical consent from the client. d. Provide emotional support to the client and significant other.

a. Assess the client's vital signs. A suspected ectopic pregnancy can put the client at risk for hypovolemic shock. The assessment of vital signs should be performed first, followed by any procedures to maintain the ABCs. Providing emotional support would also occur, as would obtaining a surgical consent, if needed, but these are not first steps.

Which finding would the nurse expect when assessing the breasts of a client with fibrocystic breast disease? a. nipple retraction b. soft mass c. enlarged lymph nodes d. skin dimpling

b. soft mass The characteristic breast mass of fibrocystic disease is soft to firm, movable, and unlikely to cause nipple retraction. Nipple retraction, enlarged lymph nodes, and skin dimpling are more commonly associated with breast cancer.

When preparing for a class on breast cancer, the nurse should explain which nonmodifiable risk factors? Select all that apply. a. BRCA1 gene b. menarche at age 11 c. being a 52-year-old female d. failing to breastfeed infants e. high-fat dietary intake

a. BRCA1 gene b. menarche at age 11 c. being a 52-year-old female Risk factors can be divided into those that cannot be changed (nonmodifiable) and that that can (modifiable). Nonmodifiable risk factors include gender, age (>50 years old), genetic mutations, personal history of ovarian or colon cancer, early menarche (<12 years old) and race (higher in white women) just to name a few. Failing to breastfeed, high fat dietary intake, and lack of exercise are examples of modifiable risk factors.

A woman with cardiac disease at 32 weeks' gestation reports she has been having spells of light-headedness and dizziness every few days. Which instruction should the nurse prioritize? a. Decrease activity and rest more often. b. Increase fluids and take more vitamins. c. Bed rest and bathroom privileges only until birth. d. Discuss induction of labor with the primary care provider.

a. Decrease activity and rest more often. If the client is developing symptoms associated with her heart condition, the first intervention is to monitor activity levels, decrease activity, and treat the symptoms. At 32 weeks' gestation, the suggestion to induce labor is not appropriate, and without knowledge of the type of heart condition one would not recommend increase of fluids or vitamins. Total bed rest may be required if the symptoms do not resolve with decreased activity.

An experienced nurse has just read that women have a one-in-three lifetime risk of developing cancer and becomes concerned that she has provided enough education. What should this nurse do to help prevent deaths from cancer? Select all that apply. a. Focus on screening b. There is nothing she can do c. Provide education to all women d. Tell the primary care providers to do a better job e. Tell women about early symptoms

a. Focus on screening c. Provide education to all women e. Tell women about early symptoms Women do have a one-in-three lifetime risk of developing cancer, and one out of every four deaths is from cancer. Therefore, nurses must focus on screening and educating all women regardless of risk factors. The other options are not acceptable.

The nurse is caring for a woman who has dysplasia (disordered growth of abnormal cells). The nurse educates her on dysplasia progression that is high-grade. Which information is important for the nurse to include? a. High-grade dysplasia progresses to invasive cervical cancer in about 2 years. b. High-grade dysplasia progresses to invasive cervical cancer in about 9 years. c. High-grade dysplasia progresses to invasive cervical cancer in about 4 years. d. High-grade dysplasia progresses to invasive cervical cancer in about 7 years.

a. High-grade dysplasia progresses to invasive cervical cancer in about 2 years. With cervical cancer, lesions start as dysplasia and progress over a period of time. Progression of a high-grade dysplasia takes about 2 years to develop into an invasive cancer.

What important instruction should the nurse give a pregnant client with tuberculosis? a. Maintain adequate hydration. b. Avoid direct sunlight. c. Avoid red meat. d. Wear light, cotton clothes.

a. Maintain adequate hydration. The nurse should instruct the pregnant client with tuberculosis to maintain adequate hydration as a health-promoting activity. The client need not avoid direct sunlight or red meat, or wear light clothes; these have no impact on the client's condition.

A female client with metastatic breast disease is receiving trastuzumab as part of her immunotherapy. The client has nausea, fatigue, diarrhea, appears jaundice, and has a distended abdomen. What would the nurse do next? a. Notify the health care provider. b. Decrease the trastuzumab infusion rate. c. Assess the client's white blood cell count. d. Continue to monitor the client.

a. Notify the health care provider. Adverse effects of trastuzumab include cardiac toxicity, vascular thrombosis, hepatic failure, fever, chills, nausea, vomiting, and pain with first infusion. The nurse should monitor for these adverse effects with the first infusion of trastuzumab. The nurse would notify the health care provider since the client is showing signs of hepatic failure.

A 24-year-old female client with a family history of cervical cancer is in the office. For which test should the nurse prepare the client to detect early stages of cervical cancer? a. Papanicolaou test b. blood test for mutations in BRCA genes c. serum CA-125 tests d. transvaginal ultrasound

a. Papanicolaou test A Papanicolaou (Pap) test should be done regularly to detect the early stages of cervical cancer. The BRCA gene is present in breast and ovarian cancers. Serum CA-125 can be a marker in ovarian cancer at 100%. Transvaginal ultrasound cannot detect early stages of cervical cancer.

A nurse is educating a 25-year-old client with a family history of cervical cancer. Which test should the nurse inform the client about to detect cervical cancer at an early stage? a. Papanicolaou test b. blood tests for mutations in the BRCA genes c. CA-125 blood test d. transvaginal ultrasound

a. Papanicolaou test The client should have Papanicolaou tests regularly to detect cervical cancer during the early stages. Blood tests for mutations in the BRCA genes indicate the lifetime risk of the client of developing breast or ovarian cancer. CA-125 is a biologic tumor marker associated with ovarian cancer, but it is not currently sensitive enough to serve as a screening tool. The transvaginal ultrasound can be used to detect endometrial abnormalities.

A 55-year-old client presents to the clinic with persistent vulvar pruritus, burning, and a lump. She states she has had the symptoms for 5 months and has been trying to treat them with over-the-counter creams. She has a history of multiple sexual partners and HPV and is a smoker. What should the nurse do next? a. Prepare the client for a biopsy of the lesion. b. Determine what creams the client has used. c. Assess how much the client smokes daily. d. Schedule the client for cryosurgery.

a. Prepare the client for a biopsy of the lesion. The client has classic symptoms of vulvar cancer. All clients with vulvar lumps should be biopsied even if they are asymptomatic. Cryosurgery may be scheduled if the biopsy confirms the diagnosis of vulvar cancer. It is not important to know what creams the client has used or how much she smokes at this time.

The postmenopausal woman who has bleeding and spotting and cannot tolerate a endometrial biopsy in the office would have which test done to rule out endometrial cancer? a. pelvic examination b. Papanicolau test c. transvaginal ultrasound d. abdominal ultrasound

c. transvaginal ultrasound If an endometrial biopsy is inconclusive for cancer, then the client would have a transvaginal ultrasound to evaluate the endometrial cavity and measure the thickness of the endometrial lining. A pelvic examination and Papanicolau test are not used to diagnose this disease.

A client presents for a routine check-up at a local health care center. One of the client's distant relatives died of ovarian cancer, and the client wants to know about measures that can reduce the risk of ovarian cancer. The nurse informs the client about which measure to reduce the risk of ovarian cancer? a. Provide genetic counseling and thorough assessment. b. Instruct the client to avoid use of oral contraceptives. c. Instruct the client to avoid breastfeeding. d. Instruct the client to use perineal talc or hygiene sprays.

a. Provide genetic counseling and thorough assessment. Only 5% of ovarian cancers are genetic in origin. However, the nurse needs to tell the client to seek genetic counseling and thorough assessment to reduce her risk of ovarian cancer. Oral contraceptives reduce the risk of ovarian cancer and should be encouraged. Breastfeeding should be encouraged as a risk-reducing strategy. The nurse should instruct the client to avoid using perineal talc or hygiene sprays.

A nurse is working as part of a community group focusing efforts on preventing isoimmunization during pregnancy at the local women's health clinic. Which measure would the group encourage? a. Rho(D) immune globulin administration to Rh-negative women b. blood typing of mothers with type A or B blood c. amniocentesis d. cerclage

a. Rho(D) immune globulin administration to Rh-negative women Rh incompatibility can be prevented with the use of Rho(D) immune globulin. Hemolysis associated with ABO incompatibility is limited to mothers with type O blood and their fetuses with type A or B blood. Amniocentesis would be appropriate for treatment of polyhydramnios not isoimmunization. Cerclage is a treatment for cervical insufficiency.

A nurse is providing education to a woman at 28 weeks' gestation who has tested positive for gestational diabetes mellitus (GDM). What would be important for the nurse to include in the client teaching? a. She is at increased risk for type 2 diabetes mellitus after her baby is born. b. Her baby is at increased risk for neonatal diabetes mellitus. c. Her baby is at increased risk for type 1 diabetes mellitus. d. She is at increased risk for type 1 diabetes mellitus after her baby is born.

a. She is at increased risk for type 2 diabetes mellitus after her baby is born. The woman who develops GDM is at increased risk for developing type 2 diabetes mellitus after pregnancy.

A woman who has sickle cell anemia asks the nurse if her infant will develop sickle cell disease. The nurse would base the answer on which information? a. Sickle cell anemia is recessively inherited. b. Sickle cell anemia has more than one polygenic inheritance pattern. c. Sickle cell anemia is dominantly inherited. d. Sickle cell anemia is not inherited; it occurs following a malaria infection.

a. Sickle cell anemia is recessively inherited. Sickle cell anemia is an autosomal recessive disease requiring that the person have two genes for the disease, one from each parent. If one parent has the disease and the other is free of the disease and trait, the chance of the child inheriting the disease is zero. If the woman has the disease and her partner has the trait, there is a 50% chance that the child will be born with the disease. If both parents have the disease, then all of their children also will have the disease.

A 1-day-old newborn is being examined by the nurse practitioner, who makes the following notation: face and sclera appear mildly jaundiced. What causes this finding? a. The breakdown of RBCs release bilirubin, which the liver cannot excrete. b. The GI tract is immature, so the bilirubin remains in the intestines. c. The newborn's Vitamin K levels are low. d. Feedings are not adequate to eliminate the build-up of bilirubin.

a. The breakdown of RBCs release bilirubin, which the liver cannot excrete. After birth, the newborn's hematocrit is about 45% to 65%, which is not needed after birth for oxygenation. The cells then die and are broken down, releasing bilirubin. The liver normally breaks down the bilirubin and eliminates it but since the liver is immature, it becomes overwhelmed and the bilirubin builds up in the bloodstream. Vitamin K levels have no effect on bilirubin levels. The immaturity of the GI tract does not cause the bilirubin to increase and feedings do not directly affect bilirubin levels.

A primigravida 21-year-old client at 24 weeks' gestation has a 2-year history of HIV. As the nurse explains the various options for delivery, which factor should the nurse point out will influence the decision for a vaginal birth? a. The viral load b. Amniocentesis results at 34 weeks c. The mother's age d. Prophylactic ART to infant at birth

a. The viral load A woman who has HIV during pregnancy is at risk for transmitting the infection to the fetus during pregnancy or childbirth and to the newborn while breastfeeding. The type of birth, vaginal or cesarean, depends on several factors, including the woman's viral load, use of ART during pregnancy (not waiting until the birth), length of time membranes have been ruptured, and gestational age (not mother's age). With prenatal ART and prophylactic treatment of the newborn, there is a reduced risk of perinatal HIV transmission. The amniocentesis results would not be a factor in preventing the spread of HIV to the infant and may actually lead to the fetus being infected through the puncture site and bleeding into the amniotic sac.

A young mother gives birth to twin boys who shared the same placenta. What serious complication are they at risk for? a. Twin-to-twin transfusion syndrome (TTTS) b. HELLP syndrome c. TORCH syndrome d. ABO incompatibility

a. Twin-to-twin transfusion syndrome (TTTS) When twins share a placenta, a serious condition called twin-to-twin transfusion syndrome (TTTS) can occur.

In which client would the nurse suspect cancer? a. a 45-year-old with thickening in one breast with nipple irritation and retraction and a pink discharge b. a 25-year-old with multiple small, round, and smooth lesions on both breasts that are painful during menstruation c. a 30-year-old with a mobile, rubbery, firm, well-circumscribed, nontender lump d. a 40-year-old with nipple retraction and a watery discharge

a. a 45-year-old with thickening in one breast with nipple irritation and retraction and a pink discharge The 25-year-old most likely has fibrocystic breast changes. The 30-year-old most likely has a fibroadenoma. The 40-year-old most likely has an intraductal papilloma. Further assessment is needed to confirm each of these, but this is what the nurse would first suspect.

A nurse practitioner is performing a clinical breast exam and notes a mass. Which findings would lead the nurse practitioner to suspect that the mass is benign? Select all that apply. a. absence of nipple retraction b. skin dimpling c. firmness on palpation d. irregularly shaped e. clearly delineated margins

a. absence of nipple retraction c. firmness on palpation e. clearly delineated margins Benign breast masses are usually firm and rubbery with clearly delineated margins and no nipple retraction. Malignant breast masses ofen are hard on palpation and irregularly shaped with skin dimpling.

A young mother has tested positive for HIV. When discussing the situation with the client, the nurse should advise the mother that she should avoid which activity? a. breastfeeding b. future pregnancies c. cesarean birth d. handling the infant with open sores

a. breastfeeding Breastfeeding is a major contributing factor for mother-to-child transmission of HIV. Cesarean birth before the onset of labor and/or rupture of membranes can greatly reduce the chance of transmitting the infection to the infant. Future pregnancies should be discussed and decided on an individual basis. Proper treatment of any open wounds and education should be provided to the mother to ensure she reduces the chance of transmitting HIV to her infant.

A nurse is caring for a 30-year-old woman who was just diagnosed with cervical cancer. Which psychosocial need would be the priority for the nurse with her client? a. clear information on the disease, management, and treatment b. touching the client's hand for comfort c. remaining cheerful through all of the interactions d. offering words of hope to the client

a. clear information on the disease, management, and treatment Women diagnosed with cancer of the reproduction tract need to understand their disease, prognosis, and what treatment options they have. The nurse's role is to educate with effective and clear communication techniques. The nurse should be sincere and may provide realistic hope, but her role as educator is primary.

A nurse prepares a woman with an abnormal Papanicolau test result for further diagnostic evaluation. Which procedure would the nurse expect to be done? a. colposcopy b. herpes simplex virus (HSV) testing c. transvaginal ultrasound d. endometrial biopsy

a. colposcopy An abnormal Papanicolau test result typically requires further evaluation, usually with a colposcopy. HSV testing would not be warranted. A transvaginal ultrasound would be used to identify endometrial thickness to determine the need for an endometrial biopsy for endometrial cancer. An endometrial biopsy would be done to evaluate for endometrial cancer.

A 29-year-old client has gestational diabetes. The nurse is teaching her about managing her glucose levels. Which therapy would be most appropriate for this client? a. diet b. long-acting insulin c. oral hypoglycemic drugs d. glucagon

a. diet Clients with gestational diabetes are usually managed by diet alone to control their glucose intolerance. Long-acting insulin usually is not needed for blood glucose control in the client with gestational diabetes. Oral hypoglycemic drugs are contraindicated in pregnancy. Glucagon raises blood glucose and is used to treat hypoglycemic reactions.

A woman with severe preeclampsia is receiving magnesium sulfate. The woman's serum magnesium level is 9.0 mEq/L. Which finding would the nurse most likely note? a. diminished reflexes b. elevated liver enzymes c. seizures d. serum magnesium level of 6.5 mEq/L

a. diminished reflexes Diminished or absent reflexes occur when a client develops magnesium toxicity, serum levels greater than 8.0 mEq/L. Elevated liver enzymes are unrelated to magnesium toxicity and may indicate the development of HELLP syndrome. The onset of seizure activity indicates eclampsia. A serum magnesium level of 6.5 mEq/L would fall within the therapeutic range of 4 to 7 mEq/L.

A woman is being closely monitored and treated for severe preeclampsia with magnesium sulfate. Which finding would alert the nurse to the development of magnesium toxicity in this client? a. diminished reflexes b. elevated liver enzymes c. seizures d. serum magnesium level of 6.5 mEq/L

a. diminished reflexes Diminished or absent reflexes occur when a client develops magnesium toxicity. Elevated liver enzymes are unrelated to magnesium toxicity and may indicate the development of HELLP syndrome. The onset of seizure activity indicates eclampsia. A serum magnesium level of 6.5 mEq/L would fall within the therapeutic range of 4 to 7 mEq/L.

The ability of the nurse to identify irregular findings during a physical assessment aids in rapid diagnosis and treatment of possible complications. The nurse assesses a newborn and notes tachycardia. The nurse notifies the health care provider based on the understanding that further assessment is necessary for which condition? a. drug withdrawal b. infection c. hypothermia d. anemia

a. drug withdrawal Tachycardia may be found with volume depletion, cardiorespiratory disease, drug withdrawal, and hyperthyroidism.

A client presents for her annual Papanicolau test. She wants to know about the risk factors that are associated with cervical cancer. Which risk factor for cervical cancer should the nurse inform the client about? a. early age at first intercourse b. obesity (at least 50 lb [22.7 kg] overweight) c. hypertension d. infertility

a. early age at first intercourse Early onset of sexual activity, within the first year of menarche, increases the risk of acquiring cervical cancer later on. Obesity, infertility, and hypertension are risk factors that are associated with endometrial cancer.

The primary care provider has just informed the client that her breast biopsy is positive for carcinoma. One of the nurse's first responsibilities is to assist with: a. education. b. treatment. c. psychiatric issues. d. telling the client her options.

a. education. The primary care provider will ultimately be responsible for treatment and deciding the client's options. The nurse will assist by helping the woman to prioritize the voluminous amount of information given to her so she can make informed decisions. The nurse should prepare an individual packet of information and review it with the client.

A nurse is conducting a refresher program for a group of perinatal nurses. Part of the program involves a discussion of HELLP. The nurse determines that the group needs additional teaching when they identify which aspect as a part of HELLP? a. elevated lipoproteins b. hemolysis c. liver enzyme elevation d. low platelet count

a. elevated lipoproteins The acronym HELLP represents hemolysis, elevated liver enzymes, and low platelets. This syndrome is a variant of preeclampsia/eclampsia syndrome that occurs in 10% to 20% of clients whose diseases are labeled as severe.

A woman with a recent incomplete abortion is to receive therapeutic misoprostol. The nurse understands that the rationale for administering this drug is to: a. ensure passage of all the products of conception. b. alleviate strong uterine cramping. c. suppress the immune response to prevent isoimmunization. d. halt the progression of the abortion.

a. ensure passage of all the products of conception. Misoprostol is used to stimulate uterine contractions and evacuate the uterus after an incomplete abortion to ensure passage of all the products of conception. Rh (D) immunoglobulin is used to suppress the immune response and prevent isoimmunization.

A woman with an incomplete abortion is to receive misoprostol. The woman asks the nurse, "Why am I getting this drug?" The nurse responds to the client, integrating understanding that this drug achieves which effect? a. ensures passage of all the products of conception b. alleviates strong uterine cramping c. suppresses the immune response to prevent isoimmunization d. halts the progression of the abortion

a. ensures passage of all the products of conception Misoprostol is used to stimulate uterine contractions and evacuate the uterus after an abortion to ensure passage of all the products of conception. Rho(D) immune globulin is used to suppress the immune response and prevent isoimmunization.

What is the most common breast mass in women? a. fibroadenoma b. mastitis c. intraductal papilloma d. duct ectasia

a. fibroadenoma A fibroadenoma is a benign mass in the breast. It is the most common breast mass among women.

A pregnant woman is diagnosed with abruptio placentae. When reviewing the woman's physical assessment in her medical record, which finding would the nurse expect? a. firm, rigid uterus on palpation b. bright red vaginal bleeding c. fetal heart rate within normal range d. absence of pain

a. firm, rigid uterus on palpation The uterus is firm-to-rigid to the touch with abruptio placentae. It is soft and relaxed with placenta previa. Bleeding associated with abruptio placentae occurs suddenly and is usually dark in color. Bleeding also may not be visible. Bright red vaginal bleeding is associated with placenta previa. Fetal distress or absent fetal heart rate may be noted with abruptio placentae. The woman with abruptio placentae usually experiences constant uterine tenderness on palpation.

A nurse is assessing a client who comes to the clinic reporting urinary incontinence. The nurse suspects that the client may be experiencing urge incontinence based on which findings? Select all that apply. a. frequency b. pain on urination c. nocturia d. small volume of urine leakage e. burning when urinating

a. frequency c. nocturia Urge incontinence is characterized by urgency, frequency, nocturia, and a large amount of urine loss. There is no pain or burning.

A client has come to the office for a prenatal visit during her 22nd week of gestation. On examination, it is noted that her blood pressure has increased to 138/90 mm Hg. Her urine is negative for proteinuria. The nurse recognizes which factor as the potential cause? a. gestational hypertension b. chronic hypertension c. HELLP d. preeclampsia

a. gestational hypertension Gestational hypertension is characterized by hypertension without proteinuria after 20 weeks of gestation resolving by 12 weeks postpartum. It is defined as systolic blood pressure of greater than 140 mm Hg and/or diastolic of greater than 90 mm Hg on at least two occasions at least 6 hours apart after the 20th week of gestation, in women known to be normotensive prior to this time and prior to pregnancy. HELLP is an acronym that refers to hemolysis, elevated liver enzymes, and low platelets. Preeclampsia may result if hypertension is not controlled or advances to a more severe state.

A nurse is providing care to a client who has been diagnosed with a common benign form of gestational trophoblastic disease. The nurse identifies this as: a. hydatidiform mole. b. ectopic pregnancy. c. placenta accrete. d. hydramnios.

a. hydatidiform mole. Gestational trophoblastic disease comprises a spectrum of neoplastic disorders that originate in the placenta. The two most common types are hydatidiform mole (partial or complete) and choriocarcinoma. Ectopic pregnancy, placenta accreta, and hydramnios fall into different categories of potential pregnancy complications.

A nurse is preparing for a presentation about breast cancer to a local women's group. Which type of breast cancer would the nurse describe as being the most common type? a. invasive ductal carcinoma b. invasive lobular carcinoma c. tubular carcinoma d. colloid carcinoma

a. invasive ductal carcinoma By far the most common breast cancer is invasive ductal carcinoma, accounting for 85% of all cases. Invasive lobular carcinoma accounts for 10% of all cases of breast cancer. Other invasive less common types are tubular carcinoma and colloid carcinoma.

A nurse is conducting a health promotion program, encouraging lifestyle changes to help clients prevent various benign and treatable conditions. Which suggestions would the nurse most likely include? Select all that apply. a. low-fat diet b. regular exercise c. high-impact aerobics d. high vegetable-fruit diet e. adequate fluid intake

a. low-fat diet b. regular exercise d. high vegetable-fruit diet e. adequate fluid intake Many of the conditions can be improved and/or prevented by following a low-fat, high vegetable-fruit diet, and getting regular exercise. High-impact aerobics should be avoided because of the excessive downward pressure placed on organs. Educating your clients will help improve their lives and possibly prevent many disorders.

The nurse is providing care to a client who has had surgery as treatment for breast cancer. The nurse would be alert for the development of which complication? a. lymphedema b. fibrocystic breast disease c. fibroadenoma d. breast abscess

a. lymphedema Lymphedema occurs in some women after breast cancer surgery. It causes disfigurement and increases the lifetime potential for infection and poor healing. Fibrocystic breast disease and fibroadenoma are two benign breast conditions that occur usually in premenopausal woman. Breast abscess is the infectious and inflammatory breast condition that is common among breastfeeding mothers.

A client is considering breast augmentation. What would the nurse recommend to the client to ensure that there are no malignancies? a. mammogram b. mastopexy c. ultrasound d. breast biopsy

a. mammogram When caring for a client considering breast augmentation, the nurse should provide her with a general guideline to have a mammogram to verify that there are no malignancies. Mastopexy involves a breast lift for drooping breasts. Ultrasound or breast biopsy would not be necessary unless there was evidence of a problem

Over the past 20 weeks, the following blood pressure readings are documented for a pregnant client with chronic hypertension: week 16 - 124/86 mm Hg; week 20 - 138/90 mm Hg; week 24 - 140/92 mm Hg; and week 28 - 142/94 mm Hg. The nurse interprets these findings as indicating which classification of her blood pressure? a. mild hypertensive b. normotensive c. prehypertensive d. severe hypertensive

a. mild hypertensive Chronic hypertension exists when the woman has high blood pressure before pregnancy or before the 20th week of gestation, or when hypertension persists for more than 12 weeks. It has been classified as normotensive (systolic less than 120 mm Hg, diastolic less than 80 mm Hg); prehypertension (systolic 120 to 139 mm Hg, diastolic 80 to 89 mm Hg); mild hypertension (systolic 140 to 159 mm Hg, diastolic 90 to 99 mm Hg); and severe hypertension (systolic 160 mm Hg or higher, diastolic 100 mm Hg or higher).

The nursing student correctly identifies vaginal cancer as one of the rarest forms of genital cancers. It is mostly asymptomatic, which makes the diagnosis even harder. What does she identify to be symptoms of this disease? Select all that apply. a. painless vaginal bleeding b. painful vaginal bleeding c. abnormal vaginal discharge d. dysuria e. constipation f. pelvic pain

a. painless vaginal bleeding c. abnormal vaginal discharge d. dysuria e. constipation f. pelvic pain Most women with vaginal cancer are asymptomatic. Those with symptoms have painlesss vaginal bleeding, abnormal vaginal discharge, dysuria, constipation, and pelvic pain.

A nurse is caring for a client undergoing treatment for ectopic pregnancy. Which symptom is observed in a client if rupture or hemorrhaging occurs before the ectopic pregnancy is successfully treated? a. phrenic nerve irritation b. painless bright red vaginal bleeding c. fetal distress d. tetanic contractions

a. phrenic nerve irritation The symptoms if rupture or hemorrhaging occurs before successfully treating the pregnancy are lower abdomen pain, feelings of faintness, phrenic nerve irritation, hypotension, marked abdominal tenderness with distension, and hypovolemic shock. Painless bright red vaginal bleeding occurring during the second or third trimester is the clinical manifestation of placenta previa. Fetal distress and tetanic contractions are not the symptoms observed in a client if rupture or hemorrhaging occurs before successfully treating an ectopic pregnancy.

A nurse is providing care to a multiparous client. The client has a history of cesarean births. The nurse anticipates the need to closely monitor the client for which condition? a. placenta accreta b. placenta abruption c. preeclampsia d. oligohydramnios

a. placenta accreta Placenta accreta is a condition where the placenta attaches itself too deeply into the wall of the uterus. According to the literature, a cesarean birth increases the possibility of a future placenta accreta; the more cesarean births that are done, the greater the incidence. Placenta abruption or abruptio placentae occurs when the placenta becomes detached prematurely. Preeclampsia and oligohydramnios are nonrelated to previous cesarean births.

A woman in labor has sharp fundal pain accompanied by slight vaginal bleeding. What would be the most likely cause of these symptoms? a. premature separation of the placenta b. preterm labor that was undiagnosed c. placenta previa obstructing the cervix d. possible fetal death or injury

a. premature separation of the placenta Premature separation of the placenta begins with sharp fundal pain, usually followed by vaginal bleeding. Placenta previa usually produces painless bleeding; labor contractions are more often described as cramping.

A woman in week 35 of her pregnancy with severe hydramnios is admitted to the hospital. The nurse recognizes that which concern is greatest regarding this client? a. preterm rupture of membranes followed by preterm birth b. development of eclampsia c. hemorrhaging d. development of gestational trophoblastic disease

a. preterm rupture of membranes followed by preterm birth Even with precautions, in most instances of hydramnios, there will be preterm rupture of the membranes because of excessive pressure, followed by preterm birth. The other answers are not as big of concerns as preterm birth, in this situation.

The nurse is teaching a premenopausal client about dietary and lifestyle modifications that can reduce the risk factors for developing pelvic organ prolapse (POP). The nurse is describing which type of prevention? a. primary b. secondary c. tertiary d. none of the above

a. primary Informing the client about factors in her lifestyle that might be modified to reduce her risk of developing POP would be an exmaple of primary prevention.

A nurse is assessing a newborn with the parents. The nurse explains that which aspect of newborn behavior is an important indication of neurologic development and function? a. reflex b. crying response c. voluntary movements d. orientation to surroundings

a. reflex The presence and strength of a reflex is an important indication of neurologic development and function. It is built into the nervous system and does not need the intervention of conscious thought to take effect. These reflexes end at different levels of the spine and brain stem, reflecting the function of the cranial nerves and motor system.

When assessing a pregnant woman with vaginal bleeding, the nurse would suspect a threatened abortion based on which finding? a. slight vaginal bleeding b. cervical dilation c. strong abdominal cramping d. passage of fetal tissue

a. slight vaginal bleeding Slight vaginal bleeding early in pregnancy, no cervical dilation, and a closed cervical os are associated with a threatened abortion. Strong abdominal cramping is associated with an inevitable abortion. With an inevitable abortion, passage of the products of conception may occur. No fetal tissue is passed with a threatened abortion.

A pregnant woman has arrived to the office reporting vaginal bleeding. Which finding during the assessment would lead the nurse to suspect an inevitable abortion? a. strong abdominal cramping b. slight vaginal bleeding c. closed cervical os d. no passage of fetal tissue

a. strong abdominal cramping Strong abdominal cramping is associated with an inevitable spontaneous abortion. Slight vaginal bleeding early in pregnancy and a closed cervical os are associated with a threatened abortion. With an inevitable abortion, passage of the products of conception may occur. No fetal tissue is passed with a threatened abortion.

A breast tumor is most likely found in what part of the breast? a. the upper outer quadrant b. near the nipple c. at the areola d. the upper inner quadrant

a. the upper outer quadrant Although they can occur throughout the breast, breast tumors are most frequently located in the upper outer quadrant of the breast.

When assessing the newborn's umbilical cord, what should the nurse expect to find? a. two smaller arteries and one larger vein b. two smaller veins and one larger artery c. one smaller vein and two larger arteries d. one smaller artery and two larger veins

a. two smaller arteries and one larger vein When inspecting the vessels in the umbilical cord, the nurse should expect to encounter one larger vein and two smaller arteries. In 0.5% of births (3.5% of twin births), there is only one umbilical artery, which can be linked to cardiac or chromosomal abnormalities.

A client was in labor for more than 36 hours and now reports urine leaking from her vagina. On examination the nurse would be inspecting for: a. urethrovaginal fistula. b. vesicovaginal fistula. c. urge incontinence fistula. d. rectovaginal fistula.

a. urethrovaginal fistula. The majority of genital fistulas are the result of obstetric trauma. When labor is obstructed or prolonged, this unrelieved compression causes ischemia, which causes pressure necrosis and subsequent fistula formation. A urethrovaginal fistula is formed between the urethra and vagina. Vesicovaginal fistulas occur between the bladder and genital tract. A rectovaginal fistula would occur between the rectum or sigmoid colon and vagina. Urge incontinence would result with the urine leaking from the urethra, not vagina.

While conducting an interview with a 38-year-old client, the nurse recognizes which factor as putting the client at the greatest risk for endometrial cancer? a. use of estrogen without progestin for hormone replacement therapy b. her age c. history of D&C; d. family history of cancer

a. use of estrogen without progestin for hormone replacement therapy One mechanism believed to be involved in the development of endometrial cancer is a history of exposure to unopposed estrogen, the cause in 75% of women. It is also uncommon before the age of 40.

A nurse is assessing a 20-year-old female. Which data finding taken during the history would indicate endometrial cancer? a. vaginal bleeding that is painless and abnormal b. diagnosis of diabetes mellitus c. diagnosis of liver disease d. severe back pain

a. vaginal bleeding that is painless and abnormal A finding of abnormal bleeding that is painless is a major sign of endometrial cancer. The diagnoses of liver disease and diabetes mellitus are risk factors for women. Back pain can be associated with many things, including ovarian cancer.

A client has been diagnosed with cervical cancer during her second trimester of pregnancy. She decides to delay treatment until after the baby is born. The nurse prepare her for birth by explaining the birth will most likely be: a. via cesarean section. b. vaginally. c. per her choice. d. induced at 37 weeks.

a. via cesarean section. In women with early-stage disease and absence of nodal involvement, there is an increasing tendency to preserve the pregnancy. The birth (when the fetal maturity is attained) should be performed using a cesarean section. Discussion with the client and her family is essential, and treatment must be individualized.

As part of a presentation on breast cancer being given to a local woman's group, the nurse describes the need for early detection through screening. Applying the guidelines from the American Cancer Society, the nurse would emphasize which recommendation? a. yearly mammograms for women over age 40 b. clinical breast examinations every 2 years for women over age 30 c. breast self-examination at least yearly for women over age 20 d. mammograms every 3 years for women between the ages of 20 and 39 years

a. yearly mammograms for women over age 40 The American Cancer Society recommends yearly mammograms for women over age 40. Clinical breast examinations are recommended every year starting at age 40. According to the American Cancer Society Breast Cancer Screening Guidelines, breast self-examination is optional.

A postmenopausal client is told at her routine gynecological exam that the primary care provider has found a cyst on her right ovary. The nurse notices that this does not cause worry for this client. What should the nurse and/or care provider tell this client? a. "You are correct not to be concerned; after all, it is only a cyst." b. "After menopause a mass on an ovary is not a cyst and should be considered cancerous until proven otherwise." c. "We will keep a eye on it and recheck it at your next yearly appointment." d. "You may get a second opinion if you would like."

b. "After menopause a mass on an ovary is not a cyst and should be considered cancerous until proven otherwise." After menopause, a mass on an ovary is not a cyst; physiologic cysts can arise only from a follicle that has not ruptured or from the cystic degeneration of the corpus luteum. Brushing it off is not responsible, and waiting a full year would put the client at serious risk from dying from the cancer. Suggesting a second opinion instead of explaining the seriousness of it to the client would not be responsible or ethical.

A pregnant client is diagnosed with syphilis. Which interviewing question would demonstrate respect for the client and therapeutic communication? a. "Why didn't you use protection when having intercourse with your partner?": b. "I am sure it is frightening to you to be diagnosed with a disease that can affect your baby." c. "I noticed that you seem fidgety. Is there something wrong besides your STI?" d. "You should have thought about what diseases you could be exposed to. At least you are HIV negative."

b. "I am sure it is frightening to you to be diagnosed with a disease that can affect your baby." The nurse needs to be supportive, empathic and accepting of the client, asking open-ended questions and acting calm and reassuring to her. By acknowledging her fears for her fetus, the nurse is demonstrating respect for her and conveying confidence that the client is trying to take care of her fetus.

The nurse is teaching a client with gestational diabetes about complications that can occur either following birth or at delivery for her baby. Which statement by the mother indicates that further teaching is needed by the nurse? a. "My baby may be very large and I may need a cesarean section to have him." b. "If my blood sugars are elevated, my baby's lungs will mature faster, which is good." c. "Beginning at 28 weeks' gestation, I will start counting with my baby's movements every day." d. "I may need an amniocentesis during the third trimester to see if my baby's lungs are ready to be born."

b. "If my blood sugars are elevated, my baby's lungs will mature faster, which is good." Elevated blood sugars delay the maturation of fetal lungs, not increase maturation time, resulting in potential respiratory distress in newborns born to diabetic mothers. Doing "kick counts", as the fetal movement monitoring is often called, is standard practice, as is the possibility of an amniocentesis to determine lung maturity during the third trimester. Health care personnel should also prepare the mother for the potential of a cesarean section delivery if the infant is too large.

While performing a clinical breast examination, the nurse notes a firm and rubbery nodule that is well circumscribed and moves freely. How should the nurse counsel the client? a. "You may have breast cancer." b. "It's most likely a fibroadenoma, but we may need to do a biopsy." c. "This is a normal breast finding, and you don't have to worry about it." d. "This could be a fibrocystic breast change, but we may need to do a biopsy."

b. "It's most likely a fibroadenoma, but we may need to do a biopsy." This description most closely matches a fibroadenoma, but diagnostic imaging and even biopsy are warranted to confirm and rule out a cancerous tumor. The nurse should never tell the client that she may have cancer because this will only cause anxiety.

A nursing mother calls the nurse and is upset. She states that her newborn son just bit her when he was nursing. Upon examining the newborn's mouth, two precocious teeth are noted on the lower central portion of the gums. What would be the nurse's best response? a. "This is most unusual! Let me get the lactation specialist to assist you in breast-feeding. It should not be a problem though." b. "Precocious teeth can occur at birth but we may need to remove them to prevent aspiration." c. "The teeth will fall out within the first month, so don't worry about them." d. "The teeth will fall out when the newborn's baby teeth come in so this is a blessing."

b. "Precocious teeth can occur at birth but we may need to remove them to prevent aspiration." Precocious or natal teeth occur infrequently but need to be addressed when they are present. They may cause the mother discomfort when nursing and pumping may be needed initially until the mother can condition the newborn not to bite. Precocious teeth are often loose and need to be removed to prevent aspiration. Even if they are not loose, they are often removed due to them causing ulcerations on the newborn's tongue from irritation. They will not just fall out and are not the newborn's actual baby teeth that are just coming in early.

The nursing instructor is teaching a student about urinary incontinence and realizes that the student needs further instruction when she makes which statement? a. "For many women with urge incontinence simple reassurance and lifestyle interventions might help." b. "Urinary incontinence is an inevitable problem of aging." c. "There are many effective treatments for urinary incontinence." d. "Urinary incontinence is not an inevitable problem of aging."

b. "Urinary incontinence is an inevitable problem of aging." There is a widespread belief that urinary incontinence is an inevitable problem of getting older and that little or nothing can be done to relieve symptoms or reverse it. This is not true. For many women with urge incontinence simple reassurance and lifestyle changes might help. If they do not, numerous effective treatments are available.

A nurse is conducting a teaching session with a group of adolescent females at a local women's health clinic. When describing appropriate screening guidelines for cervical cancer, at which age would the nurse would instruct the group to have their first Papanicolau test? a. 18 b. 21 c. 25 d. 30

b. 21 Although professional medical organizations disagree as to the recommended frequency of screening for cervical cancer, ACOG (2015) recommends that cervical cancer screening should begin at age 21 years (regardless of sexual history) since women younger than age 21 are at very low risk of cancer.

A client asks the nurse, "I've heard that eating too much red meat can increase my risk for breast cancer. So how much red meat can I eat each day?" The nurse responds, saying that the client should limit the intake of red meat to how many ounces per day? a. 2 b. 3 c. 4 d. 5

b. 3 Research suggests that individuals limit their intake of red meat to approximately 3 ounces each day.

A mother is upset because her newborn has lost 6 ounces since birth 2 days ago. The nurse informs the mother that it is normal for a newborn to lose which percentage of their birth weight within the first week of life? a. 10% to 15% of their birth weight b. 5% to 10% of their birth weight c. 15% to 18% of their birth weight d. 20% of their birth weight

b. 5% to 10% of their birth weight Adequate digestion and absorption are essential for newborn growth and development. Normally, term newborns lose 5% to 10% of their birth weight as a result of insufficient caloric intake within the first week after birth.

The nurse is reviewing with a new client the diagnosis of polycystic ovarian syndrome (PCOS). Which long-term health problems would the nurse review as a risk with this syndrome? a. migraine b. celiac disease c. type 2 diabetes d. cellulitis

c. type 2 diabetes With PCOS, the client is at risk for long-term health issues such as reproductive cancers, cardiovascular disease, hypertension, type 2 diabetes, and dyslipidemia. Migraines can be caused by many unknown factors and appear in numerous disorders. Celiac disease is not associated with PCOS. Cellulitis is an acute infection, not a long-term health issue.

A 38-year-old woman comes into the obstetrician's office for prenatal care, stating that she is about 12 weeks pregnant with her first child. What questions would the nurse ask this client, considering her age and potential sensitivity to being labeled an "older" primipara? a. Inquire about any family history of chromosomal abnormalities since older women are more likely to have infants with a chromosomal defect. b. Be non-judgmental in your history gathering and offer her pregnancy resources to read and explore. c. Offer genetic counseling and an early amniocentesis to determine if termination is needed. d. Ask the mother if she has any chronic illnesses that the doctor needs to know about due to her being older.

b. Be non-judgmental in your history gathering and offer her pregnancy resources to read and explore. Women are having babies later in life and nurses must be supportive of their choices to postpone pregnancy. Most women realize the increased risks for having a baby after 35 years of age and don't need constant reminding of all the potentially bad outcomes that can occur. The majority of pregnancies to women over 35 years of age end up with healthy babies and mothers.

Mammography is recommended for a client diagnosed with intraductal papilloma. Which factor should the nurse ensure when preparing the client for a mammography? a. Client has not consumed fluids 1 hour before testing. b. Client has not applied deodorant on the day of testing. c. Client is just going to start her menses. d. Client has taken an aspirin before the testing.

b. Client has not applied deodorant on the day of testing. When preparing a client for mammography, the nurse should ensure the client has not applied deodorant or powder on the day of testing because these products can appear on the X-ray film as calcium spots. It is not necessary for the client to avoid fluid intake 1 hour prior to testing. Mammography has to be scheduled just after the client's menses to reduce chances of breast tenderness, not when the client is going to start her menses. The client can take aspirin or acetaminophen after the completion of the procedure to ease any discomfort, but these medications are not taken before mammography.

The nurse walks into a client's room and notes a small fan blowing on the mother as she holds her infant. The nurse should explain this can result in the infant losing body heat based on which mechanism? a. Conduction b. Convection c. Radiation d. Evaporation

b. Convection There are four main ways that a newborn loses heat; convection is one of the four and occurs when cold air blows over the body of the infant resulting in a cooling to the infant. Conductive heat loss occurs when the newborn's skin touches a cold surface, causing body heat to transfer to the colder object. Heat loss occurs by radiation to a cold object that is close to, but not touching, the newborn. Evaporative heat loss happens when the newborn's skin is wet. As the moisture evaporates from the body surface, the newborn loses body heat along with the moisture. The cold air blowing on the infant's skin will cause heat loss.

A 28-year-old client with a history of endometriosis presents to the emergency department with severe abdominal pain and nausea and vomiting. The client also reports her periods are irregular with the last one being 2 months ago. The nurse prepares to assess for which possible cause for this client's complaints? a. Healthy pregnancy b. Ectopic pregnancy c. Molar pregnancy d. Placenta previa

b. Ectopic pregnancy The most commonly reported symptoms of ectopic pregnancy are pelvic pain and/or vaginal spotting. Other symptoms of early pregnancy, such as breast tenderness, nausea, and vomiting, may also be present. The diagnosis is not always immediately apparent because many women present with complaints of diffuse abdominal pain and minimal to no vaginal bleeding. Steps are taken to diagnose the disorder and rule out other causes of abdominal pain. Given the history of the client and the amount of pain, the possibility of ectopic pregnancy needs to be considered. A healthy pregnancy would not present with severe abdominal pain unless the client were term and she was in labor. With a molar pregnancy the woman typically presents between 8 to 16 weeks' gestation reporting painless (usually) brown to bright red vaginal bleeding. Placenta previa typically presents with painless, bright red bleeding that begins with no warning.

A nurse is caring for a client for whom estrogen replacement therapy has been recommended for pelvic organ prolapse. Which nursing intervention is the most appropriate for the nurse to implement before the start of the therapy? a. Discuss the effective dose of estrogen required to treat the client. b. Evaluate the client to validate her risk for complications. c. Discuss the dietary modifications following therapy. d. Discuss the cost of estrogen replacement therapy.

b. Evaluate the client to validate her risk for complications. Before starting estrogen replacement therapy, each woman must be evaluated on the basis of a thorough medical history to validate her risk for complications such as endometrial cancer, myocardial infarction, stroke, breast cancer, pulmonary emboli, or deep vein thrombosis. The effective dose of estrogen required, the dietary modifications, and the cost of estrogen replacement therapy can be discussed at a later stage when the client understands the risks associated with estrogen replacement therapy and decides to use hormone therapy.

The primary care provider has prescribed estrogen replacement therapy (ERT) for a menopausal woman who has been diagnosed with pelvic organ prolapse (POP). The client asks the nurse why she needs to be on hormones. Which would be the nurse's best response? a. Hormone replacement will decrease blood perfusion and the elasticity of the vaginal wall. b. Hormone replacement will increase blood perfusion and the elasticity of the vaginal wall. c. Hormone replacement will increase the blood perfusion and decrease the elasticity of the vaginal wall. d. Hormone replacement will decrease blood perfusion and increase the elasticity of the vaginal wall.

b. Hormone replacement will increase blood perfusion and the elasticity of the vaginal wall. Hormone replacement therapy may improve the tone and vascularity of the supporting tissue in perimenopausal and menopausal women by increasing blood perfusion and the elasticity of the vaginal wall.

A 25-year-old client at 22 weeks' gestation is noted to have proteinuria and dependent edema on her routine prenatal visit. Which additional assessment should the nurse prioritize and alert the RN or health care provider? a. Initial BP 120/80mm Hg; current BP 130/88 mm Hg b. Initial BP 100/70 mm Hg; current BP 140/90 mm Hg c. Initial BP 140/85 mm Hg; current BP 130/80 mm Hg d. Initial BP 110/60 mm Hg; current BP 112/86 mm Hg

b. Initial BP 100/70 mm Hg; current BP 140/90 mm Hg A proteinuria of trace to 1+ and a rise in blood pressure to above 140/90 mm Hg is a concern the client may be developing preeclampsia. The blood pressures noted in the other options are not indicative of developing preeclampsia. The edema would not necessarily be indicative of preeclampsia; however, edema of the face and hands would be a concerning sign for severe preeclampsia.

The nursing instructor is teaching a class on the physiologic properities involved with the birthing process. The instructor determines the session is successful when the students correctly match surfactant with which function? a. It expands the lungs with breaths. b. It keeps alveoli from collapsing with breaths. c. It removes fluid from the lungs. d. It allows oxygen to move in the lungs.

b. It keeps alveoli from collapsing with breaths. The role of surfactant is to act on surface tension and assist in keeping the alveoli open in the lungs so the lungs do not collapse with the respiratory effort of the newborn. Surfactant does not expand the lungs, remove fluid from the lungs, or allow oxygen to move in the lungs.

A client comes to the genitourinary clinic with very mild symptoms of pelvic organ prolapse (POP) that has just started in the last several days. What would be the treatment of choice for this client? a. surgery b. Kegel exercises c. nothing d. colpexin sphere

b. Kegel exercises Kegel exercises strengthen the pelvic floor muscles to support the inner organs and prevent further prolapse; they might limit the progression of mild prolapse and alleviate some symptoms. They will not, however, help severe uterine prolapse. Surgery is for more severe cases. Doing nothing is not an option, and the colpexin sphere would be used in a case that had more pronounced symptoms.

A woman is crying because she just recently received the results of her biopsies, and they confirm that she has invasive breast cancer. Which response by the nurse is the most appropriate? a. "I'm sure you are going to be fine. You are in great hands." b. Listen to the woman talk, and remain silent for a while c. "You'll beat this thing, I know it. You are very strong." d. "I know a great support group you can join."

b. Listen to the woman talk, and remain silent for a while When a woman first receives the devastating news of the diagnosis of cancer, most often the best response is to allow the woman to express her feelings and concerns before speaking. Giving her false reassurances is not therapeutic and can break reliability and trust in a provider/patient relationship. Attempting to give her information about groups or next steps before she is in a state to take it in is also nontherapeutic.

The nurse is conducting an assessment on a newborn and witnesses a startled response with the extension of the arms and legs. The nurse should document this as which response? a. Fencing b. Moro c. Tonic neck d. Rooting

b. Moro The Moro reflex is also known as the startle reflex. When the infant is startled, they extend their arms and legs away from the body. The fencing reflex is also called the tonic neck reflex and is a total body assessment. The rooting reflex assesses the infant's ability to "look" for food.

A nursing student is preparing a class for new mothers about adaptations they can expect in their newborns. Which information about newborn vision should the student include in the presentation? a. Newborns have the ability to focus only on objects far away. b. Newborns have the ability to focus only on objects in close proximity. c. Newborns have the ability to focus on objects in midline. d. Newborns cannot focus on any objects.

b. Newborns have the ability to focus only on objects in close proximity. In regards to vision the newborn has the ability to focus on objects only in close proximity (8 to 30 cm away) and tracks objects in midline or beyond. Vision is the least mature sense at birth.

A pregnant woman at 38 weeks' gestation is receiving care for preeclampsia and suddenly complains of sharp abodminal pain. Which action should the nurse prioritize if the nurse notes a firm, distended and painful abdomen and dark red vaginal bleeding? a. Implement a tocometer b. Obtain a full set of vital signs. c. Place on the fetal heart monitor. d. Dipstick the urine for protein

b. Obtain a full set of vital signs. The initial assessment is to determine if the client is going into shock from hemorrhaging. The abdominal pain and dark red bleeding indicates a possible abruption of the placenta related to the preeclampsia. The health care provider must be notified ASAP. Assessing the status of the fetus would be next. Assessing for contractions and the urine for protein are not the priority assessment for the nurse with this client.

The nurse is assessing a woman with Class III heart disease who is in for a prenatal visit. What would be the first recognizable sign that this client is in heart failure? a. Audible wheezes b. Persistent rales in the bases of the lungs c. Elevated blood pressure d. Low blood pressure

b. Persistent rales in the bases of the lungs The earliest warning sign of cardiac decompensation is persistent rales in the bases of the lungs.

A nurse working in the Family Birthing Center is answering the nurse hot line phone. A client calls in to schedule her annual Papanicolau test. How could the nurse best educate the client before her procedure to make sure results are not affected? a. Make sure the appointment is 5 days after the last menses. b. Refrain from sexual intercourse 48 hours before testing to ensure clear results. c. It is safe to use tampons 72 hours before testing. d. The client may douche at least 48 hours before testing.

b. Refrain from sexual intercourse 48 hours before testing to ensure clear results. Nurses should use teaching guidelines with clients to optimize the Papanicolau test results. Strategies to educate would include: no douche, no tampons, no jellies, no spermicides, no intercourse. The optimal time for testing is 2 weeks after the first day of your last menses.

A nurse is teaching a new mother about what to expect for bowel elimination in her newborn. Because the mother is breastfeeding, what should the nurse tell her about the newborn's stools? a. Stools should be yellow-green and loose. b. Stools should be yellow-gold, loose, and stringy to pasty. c. Stools should be greenish and formed in consistency. d. Stools should be brown and loose.

b. Stools should be yellow-gold, loose, and stringy to pasty. The stools of a breast-fed newborn are yellow-gold, loose, and stringy to pasty in consistency. The stools of the formula-fed newborn vary depending on the type of formula ingested. They may be yellow, yellow-green, or greenish and loose, pasty, or formed in consistency, and they have an unpleasant odor.

The nurse is preparing a postpartum nursing care plan for a single HIV-positive primigravida client. The nurse should prioritize in the plan how to acquire which resource? a. Breast pump b. Diapers c. Car seat d. Formula

d. Formula It is possible to transmit HIV via breastfeeding, and formula is the only option for feeding. The nurse needs to provide positive information and offer to make a referral or get assistance for clients who may be in financial need. In this case, acquiring adequate amounts of formula would be the priority. The diapers and a car seat are also necessary but would follow the formula. The client would not need a breast pump since she cannot give the milk to her baby.

The nurse teaches a woman to change tampons frequently, wash her hands thoroughly before and after inserting or removing tampons; use the lowest absorbency tampon, and store tampons correctly. The nurse should point out these actions will help avoid which disorder? a. PID b. TSS c. Endometriosis d. STI

b. TSS Toxic shock syndrome is a rare illness typically caused by bacteria. Teaching the client methods to decrease the exposure to bacteria is assisting in prevention of TSS. Pelvic inflammatory disorder occurs most commonly in association with untreated STIs, particularly in gonorrhea and chlamydia. Endometriosis is a painful reproductive and immunologic disorder in which tissue implants resembling endometrium grow outside of the uterus. Sexually transmitted infections are associated with sexual practices, not hygiene.

A nurse needs to provide preoperative care to a woman who is undergoing a scheduled hysterectomy. Her diagnosis is uterine fibroids. Which preoperative instruction should the nurse provide? a. The client should reduce her activity. b. The client should turn, cough, and deep-breathe. c. The client should avoid a high-calorie diet. d. The client should rest the pelvic area.

b. The client should turn, cough, and deep-breathe. The client should learn to turn, cough, and deep-breathe to prevent postoperative respiratory issues such as atelectasis and pneumonia. Discharge planning after surgery is completed will include instructions on resting the pelvis and decreasing activity. Regular surgery does not require a reduction in calories.

A client gives birth to a baby at a local health care facility. The nurse observes that the infant is fussy and begins to move her hands to her mouth and suck on her hand and fingers. How should the nurse interpret these findings? a. The infant is entering the habituation state. b. The infant is attempting self-consoling maneuvers. c. The infant is in a state of hyperactivity. d. The infant is displaying a state of alertness.

b. The infant is attempting self-consoling maneuvers. The hand-to-mouth movement of the baby indicates the self-quieting and consoling ability of a newborn. The other options are states of behavior of a newborn but are not applicable to this situation.

A woman arrives at the prenatal clinic and is accompanied by her partner. Which behaviors would be suggestive of intimate partner violence (IPV)? Select all that apply. a. The pregnant client looks at the examiner when asked questions. b. The partner answers questions for the pregnant client. c. The partner is overly protective of the pregnant client. d. Poor weight gain during the pregnancy and low birth weight infant e. The client asks questions of the nurse about her pregnancy.

b. The partner answers questions for the pregnant client. c. The partner is overly protective of the pregnant client. d. Poor weight gain during the pregnancy and low birth weight infant Intimate partner violence (IPV) occurs in both heterosexual as well as same-sex relationships. The nurse needs to be on the lookout for signs of violence when caring for women. It is estimated that 4% to 8% of pregnant women experience abuse during the pregnancy. Signs include a passive or quiet client who may appear unkempt or depressed. The abuser often refuses to leave the client alone with the health care providers and answers questions posed to the mother. The abuser is often overly protective of the client. Consequences of abuse include poor weight gain during the pregnancy, late entry into prenatal care, preterm labor and fetal death.

A client has been referred for a colposcopy by the primary care provider. The client wants to know more about the examination. Which information regarding a colposcopy should the nurse give to the client? a. Client may feel pain in the vaginal area during the examination. b. The test is conducted because of abnormal results in a Papanicolau test. c. Intercourse should be avoided for at least 1 week afterward. d. Client may experience pain during urination for a week following the test.

b. The test is conducted because of abnormal results in a Papanicolau test. The nurse should explain to the client that the colposcopy is done because the care provider has observed abnormalities in the Papanicolau test results. The nurse should also explain to the client that the procedure is painless and there are no adverse effects, such as pain during urination. There is no need to avoid intercourse for a week after the colposcopy.

A 47-year-old woman was just diagnosed with a cancer of her reproductive tract. The public health nurse is aiding in counseling. Which nursing interventions would be supportive in counseling this woman? Select all that apply. a. Be judgmental of the women's previous lifestyle. b. Use sincere, basic communication techniques. c. Give care based on all women diagnosed with cancer. d. Validate the client's feelings and provide realistic hope. e. Give postoperative care and instructions when prescribed.

b. Use sincere, basic communication techniques. d. Validate the client's feelings and provide realistic hope. e. Give postoperative care and instructions when prescribed. Nursing interventions in caring for women with cancers of the female reproductive tract include the following: Validate the client's feelings and provide realistic hope; use basic communication skills in a caring way; give useful, nonjudgmental information to all women; give individual care for each person; and give discharge and postoperative care when ordered.

During a follow-up visit, a female client who underwent a mastectomy asks the nurse if she can work in her backyard or at least do some household work. Which suggestion would be most appropriate? a. Avoid working in the garden or yard altogether. b. Wear gloves and protective clothing to avoid any injuries. c. Increase the frequency of follow-up visits if she does work. d. Avoid household chores for at least 6 to 9 months.

b. Wear gloves and protective clothing to avoid any injuries. The nurse should recommend that the client wear gloves when doing backyard work or housework to prevent injuries that may heal slowly or become infected. Working, whether it be in the backyard or doing some household chores, can be helpful in promoting feelings of usefulness, thereby enhancing the client's coping abilities and self-esteem. She could be advised to follow up more frequently; however, this would not help prevent any untoward injury.

An elderly woman who has not seen a health care provider in 15 years comes to the gynecology (GYN) clinic with reports of pelvic and back pain, along with weight loss which she attributes to her lack of appetite. She states that she has been very weak and fatigued for the last 12 months and attributes this to depression. What should the nurse expect this client to be diagnosed with? a. endometrial cancer b. advanced cervical cancer c. ovarian cancer d. vaginal cancer

b. advanced cervical cancer The health care worker should expect advanced cervical cancer in women with pelvic, back, or leg pain, weight loss, anorexia, weakness and fatigue, and fractures.

A woman's obstetrician prescribes vitamin K supplements for a client who is on antiepileptic medications beginning at 36 weeks' gestation. The mother asks the nurse why she is taking this medication. The nurse's best response would be: a. vitamin K helps in keeping the placenta healthy. b. antiepileptic therapy can lead to vitamin K-deficient hemorrhage of the newborn. c. administration of vitamin K aids in lung maturity of the fetus. d. The antiepileptic medications can cause the mother's platelets to drop.

b. antiepileptic therapy can lead to vitamin K-deficient hemorrhage of the newborn. Antiepileptic therapy may cause vitamin K-deficient hemorrhage of the newborn that the vitamin K injection the newborn receives following birth cannot fully correct. Therefore, some physicians recommend a Vitamin K supplement for their pregnant patients beginning at 36 weeks' gestation. If the mother should go into preterm labor, the newborn will have received the vitamin K prior to delivery. However, many physicians now question the usefulness of the prophylaxis.

A pregnant woman in her 39th week of pregnancy presents to the clinic with a vaginal infection. She tests positive for chlamydia. What would this disease make her infant at risk for? a. deafness b. blindness c. neonatal laryngeal papillomas d. chicken pox

b. blindness A pregnant woman who contracts chlamydia is at increased risk for spontaneous abortion (miscarriage), preterm rupture of membranes, and preterm labor. The postpartum woman is at higher risk for endometritis (Fletcher & Ball, 2006). The fetus can encounter bacteria in the vagina during the birth process. If this happens, the newborn can develop pneumonia or conjunctivitis that can lead to blindness.

After teaching a group of students about the signs and symptoms of breast cancer, the instructor determines that additional teaching is needed when the group identifies which sign? a. peau d'orange skin b. breast symmetry c. nipple retraction d. painless mass

b. breast symmetry The primary sign of breast cancer is a painless mass in the breast. Other signs of breast cancer include a bloody discharge from the nipple, a dimpling of the skin over the lesion, retraction of the nipple, peau d'orange (orange peel) appearance of the skin, and a difference in size between the breasts.

A nurse is monitoring a client with PROM who is in labor and observes meconium in the amniotic fluid. What does the observation of meconium indicate? a. cord compression b. fetal distress related to hypoxia c. infection d. central nervous system (CNS) involvement

b. fetal distress related to hypoxia When meconium is present in the amniotic fluid, it typically indicates fetal distress related to hypoxia. Meconium stains the fluid yellow to greenish brown, depending on the amount present. A decreased amount of amniotic fluid reduces the cushioning effect, thereby making cord compression a possibility. A foul odor of amniotic fluid indicates infection. Meconium in the amniotic fluid does not indicate CNS involvement.

The nurse is providing care to a neonate. Review of the maternal history reveals that the mother is suspected of heroin use disorder. The nurse would be alert for which finding when assessing the neonate? a. low, feeble cry b. hypertonicity c. easy consolability d. vigorous sucking

b. hypertonicity Newborns of mothers with heroin or other narcotic use disorder display irritability, hypertonicity, a high-pitched cry, vomiting, diarrhea, respiratory distress, disturbed sleeping, sneezing, diaphoresis, fever, poor sucking, tremors, and seizures.

The nurse is taking a history on a woman suspected of having uterine fibroids. Which findings in her history would the nurse recognize as predisposing factors? Select all that apply. a. smoking b. obesity c. age d. nulliparity d. hyperinsulinemia

b. obesity c. age d. nulliparity Predisposing factors for uterine fibroids include the following: age, nulliparity, obesity, genetic predisposition, and African American ethnicity. A history of smoking and a diagnosis of hyperinsulinemia are not factors for risk.

A nursing student is studying gynecological cancers and is excited when she reads that birth control pills can have a positive effect on preventing which disease? a. uterine cancer b. ovarian cancer c. endometrial cancer d. vaginal cancer

b. ovarian cancer Risk-reduction strategies for preventing ovarian cancer include pregnancy, use of oral contraceptives, and breastfeeding. No research states that oral contraceptives help to prevent the other cancers.

A pregnant client has an Rh-negative blood type. Following the birth of the client's infant, the nurse administers her Rho(D) immune globulin. The purpose of this is to: a. promote maternal D antibody formation. b. prevent maternal D antibody formation. c. stimulate maternal D immune antigens. d. prevent fetal Rh blood formation.

b. prevent maternal D antibody formation. Because Rho(D) immune globulin contains passive antibodies, the solution will prevent the woman from forming long-lasting antibodies.

A 24-year-old client presents in labor. The nurse notes there is an order to administer RhoGAM after the birth of her infant. When asked by the client the reason for this injection, which reason should the nurse point out? a. promote maternal D antibody formation. b. prevent maternal D antibody formation. c. stimulate maternal D immune antigens. d. prevent fetal Rh blood formation.

b. prevent maternal D antibody formation. Because RhoGAM contains passive antibodies, the solution will prevent the woman from forming long-lasting antibodies which may harm a future fetus. The administration of RhoGAM does not promote the formation of maternal D antibodies; it does not stimulate maternal D immune antigens or prevent fetal Rh blood formation.

A nurse is conducting a presentation for a local women's group about pelvic organ prolapse. When describing the different types, which information would the nurse incorporate into the description of a cystocele? a. sagging of the rectum, pushing against or into the posterior vaginal wall b. protrusion of the bladder wall through the anterior vaginal wall c. bulging of the small intestine through the posterior vaginal wall d. downward movement of the uterus through the pelvic floor and into the vagina

b. protrusion of the bladder wall through the anterior vaginal wall A cystocele occurs when the posterior bladder wall protrudes downward through the anterior vaginal wall. A rectocele occurs when the rectum sags and pushes into or against the posterior vaginal wall. An enterocele occurs when the small intestine bulges through the posterior vaginal wall. Uterine prolapse occurs when the uterus descends through the pelvic floor and into the vaginal canal.

A nurse is performing a nursing assessment on a new client. The woman estimates that she is approximately 16 weeks pregnant. While assessing her the nurse asks her about what appear to be scratch marks on her hands, and she tells the nurse that she has three cats at home. What screening would be prescribed for this woman? a. cytomegalovirus b. toxoplasmosis c. hepatitis C d. herpes simplex virus

b. toxoplasmosis Toxoplasmosis is an infection caused by the protozoan Toxoplasma gondii, also referred to as T. gondii. Transmission is via undercooked meat and through cat feces. Toxoplasmosis is a common infection in humans and usually produces no symptoms. However, when the infection passes from the woman through the placenta to the fetus, a condition called congenital toxoplasmosis can occur. Approximately 400 to 4,000 cases of congenital toxoplasmosis occur per year in the United States (Williams, 2007). The classic triad of symptoms for congenital toxoplasmosis is chorioretinitis, intracranial calcification, and hydrocephalus in the newborn.

A nurse is caring for a client who has been prescribed gonadotropin-releasing hormone (GnRH) medication for uterine fibroids. For which side effect of GnRH medications should the nurse monitor the client? a. increased vaginal discharge b. vaginal dryness c. urinary tract infections d. vaginitis

b. vaginal dryness Vaginal dryness is one of the side effects of GnRH medications. The other side effects of GnRH medications are hot flashes, headaches, mood changes, musculoskeletal malaise, bone loss, and depression. Increased vaginal discharge, urinary tract infections, and vaginitis are side effects of a pessary, not GnRH medications.

A woman has been given the diagnosis of uterine fibroids. Prescribed treatment consists of gonadotropin-releasing hormone (GnRH). The nurse is teaching her client about the side effects of this medication. Which side effect would be important to include in the explanation? a. increased vaginal discharge b. vaginal dryness c. urinary tract infection d. vaginitis

b. vaginal dryness Side effects of GnRH include hot flashes, vaginal dryness, headaches, mood changes, depression, bone loss, and musculoskeletal malaise. Other side effects require use of a pessary, a hard plastic or rubber device placed in the vagina for support.

When assessing a client, a nurse determines that the client has a rectocele based on which finding? a. protrusion of the bladder wall through the anterior vaginal wall b. bulging of the small intestine through the posterior vaginal wall c,. sagging of the rectum, which pushes against or into the posterior vaginal wall d. downward movement of the uterus through the pelvic floor and into the vagina

c,. sagging of the rectum, which pushes against or into the posterior vaginal wall A rectocele occurs when the rectum sags and pushes into or against the posterior vaginal wall. A cystocele occurs when the posterior bladder wall protrudes downward through the anterior vaginal wall. An enterocele occurs when the small intestine bulges through the posterior vaginal wall. Uterine prolapse occurs when the uterus descends through the pelvic floor and into the vaginal canal.

A nursing instructor is teaching students about preexisting illnesses and how they can complicate a pregnancy. The instructor recognizes a need for further education when one of the students makes which statement? a. "A pregnant woman with a chronic condition can put herself at risk." b. "A pregnant woman with a chronic illness can put the fetus at risk." c. "A pregnant woman does not have to worry about contracting new illnesses during pregnancy." d. "A pregnant woman needs to be careful of and cautious about accidents and illnesses during her pregnancy."

c. "A pregnant woman does not have to worry about contracting new illnesses during pregnancy." When a woman enters a pregnancy with a chronic illness, it can put both her and the fetus at risk. She needs to be cautious about developing a new illness during her pregnancy as well as having an accident during the pregnancy.

A client at 11 weeks' gestation experiences pregnancy loss. The client asks the nurse if the bleeding and cramping that occurred during the miscarriage were caused by working long hours in a stressful environment. What is the most appropriate response from the nurse? a. "Your spontaneous bleeding is not work-related." b. "It is hard to know why a woman bleeds during early pregnancy." c. "I can understand your need to find an answer to what caused this. Let's talk about this further." d. "Something was wrong with the fetus."

c. "I can understand your need to find an answer to what caused this. Let's talk about this further." Talking with the client may assist her to explore her feelings. She and her family may search for a cause for a spontaneous early bleeding so they can plan for future pregnancies. Even with modern technology and medical advances, however, a direct cause cannot usually be determined.

The nurse is teaching a client with mastitis about care measures. Which client statement indicates effective teaching? a. "I need to wear a breast shield to keep the area moist." b. "I can stop the antibiotic after 5 days." c. "I can use warm soaks to my breast to relieve the discomfort." d. "I should avoid pumping my breasts until the infection is cleared."

c. "I can use warm soaks to my breast to relieve the discomfort." Applying warm soaks to the breast or letting warm water from the shower flow over the breast can help to relieve some of the discomfort. Breast shields should be avoided because they trap breast milk and moisture around the nipple. The client needs to continue antibiotic therapy as prescribed for the entire treatment period, usually 10 days. The client can express milk with a breast pump until the infection resolves sufficiently to resume breast feeding.

A nurse is preparing a client for discharge following an abdominal hysterectomy for fibroids. After providing discharge teaching, the nurse determines that the teaching was successful based on which client statement? a. "I can resume using tampons in about 3 weeks." b. "I don't have any restrictions on what I can lift or carry." c. "I should shower rather than take a tub bath." d. "I should limit my intake of foods that are high in fiber for about 2 weeks."

c. "I should shower rather than take a tub bath." Following an abdominal hysterectomy, the client should not insert anything in her vagina for about 6 weeks (pelvic rest), should avoid heavy lifting or straining for about 6 weeks, shower rather than take tub baths, and increase her intake of high-fiber foods to promote bowel elimination and prevent straining.

Which statement is true regarding fetal and newborn senses? a. A newborn cannot experience pain. b. A newborn cannot see until several hours after birth. c. A newborn does not have the ability to discriminate between tastes. d. The rooting reflex is an example that the newborn has a sense of touch. e. A fetus is unable to hear in utero.

d. The rooting reflex is an example that the newborn has a sense of touch. Newborns experience pain, have vision, and can discriminate between tastes. The rooting reflex is an example of a newborn's sense of touch. The fetus can hear in utero.

A 32-year-old woman with epilepsy mentions to the nurse during a routine well-visit that she would like to have children and asks the nurse for advice. Which response is most appropriate from the nurse? a. "You should talk to the doctor about that; the medications you're on can damage the fetus." b. "Do you want to talk to a counselor who can help you weigh the pros and cons of having your own child rather than adopting?" c. "I'll let the doctor know so you can discuss your medications. In the meantime, I'll give you a list of folate-rich foods you can add to your diet." d. "That's great. I've got a 4-year-old and a 2-year-old myself."

c. "I'll let the doctor know so you can discuss your medications. In the meantime, I'll give you a list of folate-rich foods you can add to your diet." Any woman with epilepsy needs to discuss the medication management with her provider. The current research indicates the medications used for epileptic management are the major cause of birth defects for these patients. The nurse should be careful about mentioning that some epileptics are teratogenic; some women may stop taking their medications in order to get pregnant. Suggesting adoption is inappropriate as the mother has given no indication she is interested in adoption; also, the mother needs to discuss this with the physician so that she can get accurate information about being on anti-seizure medications and being pregnant. The nurse should not share personal information as it does not assist this client in making a serious decision. The client should be referred to the health care provider to help the client make the best decision.

A new mother of a newborn girl calls the clinic in a panic, concerned about the blood-tinged soiled diaper. What is the best response from the nurse? a. "The baby may have a problem; let's schedule an appointment." b. "This can be related to cleaning her perineal area; be more careful." c. "This can be from the sudden withdrawal of your hormones. It is not a cause for alarm." d. "If this continues, call us back; for now, just watch her."

c. "This can be from the sudden withdrawal of your hormones. It is not a cause for alarm." The mother is describing pseudomenstruation and is usually the result of the infant no longer having the mother's hormones in the body. This is not a cause for alarm. It is always appropriate to offer to schedule an appointment if the mother continues to be upset. The nurse should know that the infant's "bleeding" is not indicative of a pathologic process and should be careful to not upset the mother further. The statement of it being related to the way the mother is cleaning the perineum is incorrect for it places the blame on the mother for the infant's problem. The instruction to call back if it continues does not meet the mother's need to know why this is happening to her baby, and it negates her concern for her infant.

A pregnant woman with diabetes is having her glycosylated hemoglobin level evaluated. The nurse determines that the woman's glucose is under control and continues the woman's plan of care based on which result? a. 8.5% b. 8.0% c. 6.5% d. 7.5%

c. 6.5% A glycosylated hemoglobin level of less than 7% indicates good control; a value of more than 8% indicates poor control and warrants intervention. A glycosylated hemoglobin level of more than 8.0% indicates poor blood glucose control and the need for intervention, necessitating a revision in the woman's plan of care.

A student nurse is reviewing newborn physical measurements and asks the charge nurse if her client's weight of 2800 g and length of 51 cm falls within normal parameters. The charge nurse would respond to the student nurse in which manner? a. A birth weight between 2200 and 3000 g is considered small for gestational age. b. A length between 48 and 50 cm plots out at the 95th percentile for length. c. A birth weight of 2800 g falls within the normal weight parameters for a full-term newborn. d. Normal birth length is usually 52 cm or above for a full-term newborn.

c. A birth weight of 2800 g falls within the normal weight parameters for a full-term newborn. Average birth weight for a newborn is between 5 lb, 8 oz (2500 g) and 8 lb, 13 oz. (4000 g). Average length at birth for a newborn is between 19 and 21 inches (48 to 53 cm).

Which intervention would be the best way for the nurse to prevent heat loss in a newborn while bathing? a. Limit the bathing time to 5 minutes. b. Bathe the baby in water between 90 and 93 degrees. c. Bathe the baby under a radiant warmer. d. Postpone breastfeeding until after the initial bath.

c. Bathe the baby under a radiant warmer. Bathing a newborn under a radiant warmer helps to prevent heat loss. To minimize the effects of cold stress during the bath, the nurse should also prewarm blankets, dry the child completely to prevent heat loss from evaporation, encourage skin-to-skin contact with the mother, promote early breastfeeding, used heated and humidified oxygen, and defer bathing until the newborn is medically stable. Limiting the length of time spent bathing the baby is secondary to maintaining the baby's body temperature. Having warm water is also important but is irrelevant if the baby is not kept warm under a warmer.

A newborn's vital signs are documented by the nurse and are as follows: HR 144, RR- 36, BP- 128/78, and T- 98.6℉ (37℃). Which finding would be concerning to the nurse? a. Heart Rate b. Respiratory Rate c. Blood Pressure d. Temperature

c. Blood Pressure The blood pressure of a newborn should be quite low—around 60-70 over 35 to 50. The heart rate and respiratory rate are both high, which are normal findings. The temperature falls within a normal range of 97.7℉ to 99.5℉ (36.5℃ to 37.5℃).

The parents are concerned their newborn appears to be cold all the time. The nurse should point out the infant is best helped by which primary method in the first few days? a. External with blankets by the nursing staff b. Skin to skin contact with mother c. Brown fat store usage d. Shivering and increased metabolic rate

c. Brown fat store usage Brown fat stores are used by the newborn infant to maintain warmth until feeding begins and the infant is able to maintain temperature without assistance. The infant's thermoregulatory system is not fully functional at birth. Infants cannot shiver to warm themselves. The use of external blankets as well as skin to skin contact with the mother assists in keeping the baby's temperature within the normal range, but they are not the primary mechanism for temperature regulation in the newborn infant.

Upon assessing the newborn's respirations, which finding would cause the nurse to notify the primary care provider? a. coughing and sneezing in the newborn b. short periods of apnea that last 10 seconds in a pink newborn c. a respiratory rate of 15 breaths per minute with nasal flaring d. a respiratory rate of 45 breaths per minute with acrocyanosis

c. a respiratory rate of 15 breaths per minute with nasal flaring Coughing and sneezing are normal reflexes present in the newborn. The respiratory rate of a newborn should be between 30 and 60 breaths per minute. Acrocyanosis can be a normal finding in a newborn and does not indicate respiratory distress. Short periods of apnea that last longer than 15 seconds in the absence of cyanosis can be normal. Nasal flaring is a sign of respiratory distress.

A nurse is conducting a class at a women's clinic about reproductive cancers. When describing the incidence of reproductive tract cancers in pregnant women, which information would the nurse include? a. Ovarian cancer is detected much later in the pregnant woman because of the hormonal changes that are occurring. b. Many cases of endometrial cancer are detected in pregnant women because of the increase in surveillance. c. Cervical cancer is more common in the pregnant population than other reproductive cancers. d. Reproductive cancers overall are more common in pregnant women.

c. Cervical cancer is more common in the pregnant population than other reproductive cancers. Cervical cancer is more common in the pregnant population, affecting the health of the woman and her fetus. Ovarian cancer occurring during pregnancy is found at early stages and is associated with a good prognosis for both the mother and newborn. Few cases of endometrial cancer would be detected during the relatively young pregnancy population since routine screening is currently not recommended in the general population. Reproductive tract cancers can occur in a pregnant woman, but their incidence is highly variable.

A woman in her 20s has experienced a miscarriage at 10 weeks' gestation and asks the nurse at the hospital what went wrong. She is concerned that she did something that caused her to lose her baby. The nurse can reassure the woman by explaining that the most common cause of spontaneous miscarriage in the first trimester is related to which factor? a. Exposure to chemicals or radiation b. Advanced maternal age c. Chromosomal defects in the fetus d. Faulty implantation

c. Chromosomal defects in the fetus Fetal factors are the most common cause of early miscarriages, with chromosomal abnormalities in the fetus being the most common reason. This client fits the criteria for early miscarriage since she was only 10 weeks pregnant and early miscarriage occurs before 12 weeks.

A nurse is reviewing the history, physical exam, and diagnostic test findings for a woman diagnosed with endometrial cancer. The findings reveal that the cancer has spread to the cervix and other parts of the uterus and to nearby lymph nodes. The nurse interprets these findings as suggestive of which stage? a. I b. II c. III d. IV

c. III In stage I, the tumor is confined to the corpus uteri. In stage II, it has spread to the cervix, but not outside the uterus. In stage III, it has spread locally (to other parts of the uterus) and regionally (to nearby lymph nodes). In stage IV, it has invaded the bladder mucosa, bowel with distant metastases to the lungs, liver, and bone.

The nurse is evaluating the morning blood glucose results from the laboratory of several 1-day-old infants. Which result should the nurse prioritize for further action? a. Infant A - 52 mg/dL b. Infant B - 56 mg/dL c. Infant C - 48 mg/dL d. Infant D - 60 mg/dL

c. Infant C - 48 mg/dL Blood glucose levels between 50 and 60 mg/dL during the 24 hours of life are considered normal. Levels less than 50 are indicative of hypoglycemia in the newborn. Infant C is showing potential hypoglycemia. Infants A, B, and D have values within the normal range.

A woman with class II heart disease is experiencing an uneventful pregnancy and is now prescribed bed rest at 36 weeks' gestation by her health care provider. The nurse should point out that this is best accomplished with which position? a. Lie flat on her back. b. Stay in high Fowler's position. c. Lie in a semi-recumbent position. d. Use pillows and wedges to stay in a fully recumbent position.

c. Lie in a semi-recumbent position. Semi-recumbent position is the best position for circulation of the mother and fetus. Lying flat on the back can induce supine hypotensive syndrome and fully recumbent impedes other circulation. The high Fowler's position would not be comfortable for sleeping, as well as possibly impede the blood flow through the hips and lower abdomen.

The nurse is caring for a pregnant client with fallopian tube rupture. Which intervention is the priority for this client? a. Monitor the client's beta-hCG level. b. Monitor the mass with transvaginal ultrasound. c. Monitor the client's vital signs and bleeding. d. Monitor the fetal heart rate (FHR).

c. Monitor the client's vital signs and bleeding. A nurse should closely monitor the client's vital signs and bleeding (peritoneal or vaginal) to identify hypovolemic shock that may occur with tubal rupture. Beta-hCG level is monitored to diagnose an ectopic pregnancy or impending abortion. Monitoring the mass with transvaginal ultrasound and determining the size of the mass are done for diagnosing an ectopic pregnancy. Monitoring the FHR does not help to identify hypovolemic shock.

A 32-year-old gravida 3 para 2 at 36 weeks' gestation comes to the obstetric department reporting abdominal pain. Her blood pressure is 164/90 mm/Hg, her pulse is 100 beats per minute, and her respirations are 24 per minute. She is restless and slightly diaphoretic with a small amount of dark red vaginal bleeding. What assessment should the nurse make next? a. Check deep tendon reflexes. b. Measure fundal height. c. Palpate the fundus, and check fetal heart rate. d. Obtain a voided urine specimen, and determine blood type.

c. Palpate the fundus, and check fetal heart rate. The classic signs of abruptio placentae are pain, dark red vaginal bleeding, a rigid, board-like abdomen, hypertonic labor, and fetal distress.

A client reports lumpy, tender breasts, particularly during the week before menses. She reports pain that often dissipates after the onset of menses. The nurse suspects the client has fibrocystic breast changes. Which should the nurse do next? a. Determine if the client has had a mammography. b. Have the client follow up in 1 week. c. Perform a breast examination. d. Schedule the client for cryoablation.

c. Perform a breast examination. To determine if the client is experiencing fibrocystic breast changes, the nurse must first examine the client's breasts. It is not important to know if the client has a mammography at this time. Cryoabation is done to remove a tumor.

A nurse is assessing the temperature of a newborn using a skin temperature probe. Which point should the nurse keep in mind while taking the newborn's temperature? a. Ensure that the newborn is lying on its abdomen. b. Tape the temperature probe on the forehead. c. Place the temperature probe over the liver. d. Use the skin temperature probe only in open bassinets.

c. Place the temperature probe over the liver. The nurse should place the temperature probe over the newborn's liver. Skin temperature probes should not be placed over a bony area like the forehead or used in an open bassinet with no heat source. The newborn should be in a supine or side-lying position.

The nurse is assessing a newborn by auscultating the heart and lungs. Which natural phenomenon will the nurse explain to the parents is happening in the cardiovascular system? a. Oxygen is exchanged in the lungs. b. Fluid is removed from the alveoli and replaced with air. c. Pressure changes occur and result in closure of the ductus arteriosus. d. The oxygen in the blood decreases.

c. Pressure changes occur and result in closure of the ductus arteriosus. The ductus arteriosus is one of the openings through which there was fetal circulation. At birth, or within the first few days, this closes and the heart becomes the source of movement of blood to and from the lungs. The exchange of oxygen in the lungs and increasing oxygen content in the blood are respiratory functions. The removal of the fluid from the alveoli occurs mainly during the birthing process and is completed by the lungs after birth.

The following hourly assessments are obtained by the nurse on a client with preeclampsia receiving magnesium sulfate: 97.3oF (36.2oC), HR 88, RR 1, BP 148/110 mm Hg. What other priority physical assessments by the nurse should be implemented to assess for potential toxicity? a. Lung sounds b. Oxygen saturation c. Reflexes d. Magnesium sulfate level

c. Reflexes Reflex assessment is part of the standard assessment for clients on magnesium sulfate. The first change when developing magnesium toxicity may be a decrease in reflex activity. The health care provider needs to be notified immediately. A change in lung sounds and oxygen saturation are not indicative of magnesium sulfate toxicity. Hourly blood draws to gain information on the magnesium sulfate level are not indicated.

The nurse is appraising the medical record of a pregnant client who is resting in a darkened room and receiving betamethasone and magnesium sulfate. The nurse will continue to monitor this client for which condition? a. Gestational hypertension b. Eclampsia c. Severe preeclampsia d. Mild preeclampsia

c. Severe preeclampsia This woman is in severe preeclampsia and must be monitored for progression to eclampsia. The administration of magnesium sulfate is to relax the skeletal muscles and raise the threshold for a seizure. The administration of the betamethasone is to try and hasten the maturity of the fetus's lungs for birth. The client has already progressed from mild preeclampsia to severe preeclampsia and measures need to be followed to prevent advancement of the disease process. The scenario described does not indicate a client with hypertension.

A client is waiting for the results of an endometrial biopsy for suspected endometrial cancer. She wants to know more about endometrial cancer and asks the nurse about the available treatment options. Which treatment information should the nurse give the client? a. Surgery involves removal of the uterus only. b. In advanced cancers, radiation and chemotherapy are used instead of surgery. c. Surgery involves removal of the uterus, fallopian tubes, and ovaries; adjuvant therapy is used if relevant. d. Follow-up care after the relevant treatment should last for at least 6 months after the treatment.

c. Surgery involves removal of the uterus, fallopian tubes, and ovaries; adjuvant therapy is used if relevant. The nurse should inform the client that surgery most often involves removal of the uterus (hysterectomy) and the fallopian tubes and ovaries (salpingo-oophorectomy). Removal of the tubes and ovaries, not just the uterus, is recommended because tumor cells spread early to the ovaries, and any dormant cancer cells could be stimulated to grow by ovarian estrogen. In advanced cancers, radiation and chemotherapy are used as adjuvant therapies to surgery. Routine surveillance intervals for follow-up care are typically every 3 to 4 months for the first 2 years.

A woman with ovarian cancer has been told that she is in stage III of the cancer. The nurse is reviewing the information with her. Which statement would help in the woman's understanding of stage III ovarian cancer? a. The cancer is limited to the ovaries. b. The growth involves one or both ovaries. c. The growth has spread to the lymph nodes and other areas/organs in the abdominal cavity. d. The cancer has spread to distant sites.

c. The growth has spread to the lymph nodes and other areas/organs in the abdominal cavity. The staging and diagnosis is performed by a laparoscopy. The staging is I to IV. Stage III means the cancer has spread to the lymph nodes and other areas in the abdominal cavity. A five-year survival rate for this stage is 30% to 60%.

The nurse prepares to give the first bath to a newborn and notes a white cheese-like substance on the skin. The nurse should document this as which substance? a. Lanugo b. Milia c. Vernix d. Amniotic fluid

c. Vernix Vernix is the coating on the infant that was covering fetal skin to prevent the skin from the drying effects of amniotic fluid. Lanugo is fine, downy hair that is present in abundance on the preterm infant but is found in thinning patches on the shoulders, arms, and back of the term newborn. Milia are frequently found on the infant's face. These tiny white papules resemble pimples in appearance. Normal amniotic fluid is not thick and white; it should be clear and give the baby a wet appearance.

A newborn has a 5-minute Apgar score of 9. What intervention should the nurse take for this client? a. Actively stimulate the infant to cry. b. Offer blow-by oxygen. c. Wrap the infant in a blanket and hand to the mother for bonding. d. Place the infant in a warmer bed and heat the newborn up.

c. Wrap the infant in a blanket and hand to the mother for bonding. Apgar scores of 7-10 at 5 minutes of age indicate a newborn is adapting well to extrauterine life and can be safely placed with the mother. A 5-minute Apgar score of 4-6 would mean that the newborn might have respiratory distress and need oxygen or requires more vigorous stimulation. Hypothermia can also cause distress and lower the Apgar score.

The nurse would expect which client to be at a high risk for developing a pelvic support disorder? a. an 18-year-old college freshman b. a 29-year-old mother of one son c. a 60-year-old mother with four children d. a 30-year-old who just gave birth to twin girls

c. a 60-year-old mother with four children Women may experience pelvic support disorders related to pelvic relaxation or urinary continence. These disorders usually develop after years of wear and tear on the muscles and tissues that support the pelvic floor, such as what occurs with childbearing, chronic coughing, straining, surgery, or simply aging.

A pregnant woman is admitted to the hospital with a diagnosis of placenta previa. Which action would be the priority for this woman on admission? a. performing a vaginal examination to assess the extent of bleeding b. helping the woman remain ambulatory to reduce bleeding c. assessing fetal heart tones by use of an external monitor d. assessing uterine contractions by an internal pressure gauge

c. assessing fetal heart tones by use of an external monitor Not disrupting the placenta is a prime responsibility. An internal monitor, a vaginal examination, and remaining ambulatory could all do this and thus are contraindicated.

A nurse will be speaking at a local high school about women's health. The nurse is planning to talk about sexually transmitted infections (STIs) as well as routine checks, along with guidelines for Papanicolau testing. What should the nurse include in the Papanicolau test guidelines about when to have a first test? a. at the age of 16 b. 2 years after first sexual intercourse c. at the age of 21 or within 3 years of first sexual intercourse d. at the age of 18 or within 2 years of first sexual intercourse

c. at the age of 21 or within 3 years of first sexual intercourse Amercian Cancer Society guidelines for Papanicolau testing recommend that the first Papanicolau test is done at age 21 or within 3 years of first sexual intercourse. Other guidelines state that the tests should be done yearly until age 30 using the glass slide method and every 2 years using liquid-based method. At age 30 to 70, the tests should be done every 2 to 3 years if the previous three Papanicolau tests were normal. They may be discontinued after age 70 if the previous three Papanicolau tests were normal and no Papanicolau tests in the previous 10 years were abnormal.

A woman is admitted with a diagnosis of ectopic pregnancy. For which procedure should the nurse prepare? a. bed rest for the next 4 weeks b. intravenous administration of a tocolytic c. immediate surgery d. internal uterine monitoring

c. immediate surgery Ectopic pregnancy means an embryo has implanted outside the uterus, usually in the fallopian tube. Surgery is usually necessary to remove the growing structure before the tube ruptures or to repair the tube if rupture has occurred already.

A 62-year-old female client arrives at a health care facility reporting skin redness in the breast area, along with skin edema. The primary care provider suspects inflammatory breast cancer. For which symptom of inflammatory breast cancer should the nurse assess? a. palpable mobile cysts b. palpable papilloma c. increased warmth of the breast d. induced nipple discharge

c. increased warmth of the breast Skin edema, redness, and warmth of the breast are symptoms of inflammatory breast cancer. Induced discharge is an indication of benign breast conditions, which are noncancerous. Cancer involves spontaneous nipple discharge. Papillomas and palpable mobile cysts are characteristics of fibroadenomas, intraductal papilloma, and mammary duct ectasia, which are benign breast conditions and are noncancerous.

The nurse is admitting a 42-year-old client to the women's medical floor. During the history, the chief report from the client is "I feel like there is a lump in my vagina, and it feels like it is sometimes dragging." Which disorder does the nurse suspect? a. urinary incontinence b. endocervical polyps c. pelvic organ prolapse d. uterine fibroids

c. pelvic organ prolapse In pelvic organ prolapse, the pelvic floor muscles are weak, and this results in a feeling of "dragging," or that some sort of "lump" is present in the vagina. Symptoms of pelvic organ prolapse do not include urinary incontinence. Endocervical polyps manifest with abnormal vaginal bleeding. With uterine fibroids, the uterus is large and has an odd shape.

A 40-year-old client arrives at the community health center experiencing a strange, dragging feeling in the vagina. She stated that "at times it feels as if there is a lump" there as well. Which condition do these symptoms indicate? a. urinary incontinence b. endocervical polyps c. pelvic organ prolapse d. uterine fibroids

c. pelvic organ prolapse Weakening of the pelvic-floor muscles causes a feeling of dragging and a "lump" in the vagina; these are symptoms of pelvic organ prolapse. These symptoms do not indicate urinary incontinence, endocervical polyps, or uterine fibroids. Urinary incontinence is the involuntary loss of urine. The symptoms of endocervical polyps are abnormal vaginal bleeding or discharge. In cases of uterine fibroids, the uterus is enlarged and irregularly shaped.

A 40-year-old woman comes to the clinic reporting having missed her period for two months. A pregnancy test is positive. What is she and her fetus at increased risk for? a. type 2 diabetes mellitus b. type 1 diabetes mellitus c. placental abnormalities d. postterm birth

c. placental abnormalities A woman older than 35 years is more likely to conceive a child with chromosomal abnormalities, such as Down syndrome. She is also at higher risk for spontaneous abortion (miscarriage), preeclampsia-eclampsia, gestational diabetes, preterm birth, bleeding and placental abnormalities, and other intrapartum complications.

A 44-year-old client has lost several pregnancies over the last 10 years. For the past 3 months, she has had fatigue, nausea, and vomiting. She visits the clinic and takes a pregnancy test; the results are positive. Physical examination confirms a uterus enlarged to 13 weeks' gestation; fetal heart tones are heard. Ultrasound reveals that the client is experiencing some bleeding. Considering the client's prenatal history and age, what does the nurse recognize as the greatest risk for the client at this time? a. premature birth b. hypertension c. pregnancy loss d. preterm labor

c. pregnancy loss The client's advanced maternal age (pregnancy in a woman 35 years or older) increases her risk for pregnancy loss. Hypertension, preterm labor, and prematurity are risks as this pregnancy continues. Her greatest risk at 13 weeks' gestation is losing this pregnancy.

A nurse working in the neonatal nursery anticipates the primary care provider to prescribe which medication for a premature newborn having difficulty breathing? a. epinephrine b. albuteral c. surfactant d. norepinephrine

c. surfactant Surfactant is a protein that keeps small air sacs in the lungs from collapsing. Its use was introduced in 1990 and continues today, especially for premature babies and those who have respiratory distress syndrome. The other medications are not given to help premature babies breathe.

A pregnant woman in her second trimester comes to the prenatal clinic for a routine visit. She reports that she has a new kitten. The nurse would have the woman evaluated for which infection? a. cytomegalovirus b. parvovirus B19 c. toxoplasmosis d. herpes simplex virus

c. toxoplasmosis Toxoplasmosis is transferred by hand to mouth after touching cat feces while changing the litter box or through gardening in contaminated soil. Cytomegalovirus is transmitted via sexual contract, blood transfusions, kissing, and contact with children in daycare centers. Parvovirus B19 is a common self-limiting benign childhood virus that causes fifth disease. A pregnant woman may transmit the virus transplacentally to her fetus if she is exposed to an infected child. Herpesvirus infection occurs by direct contact of the skin or mucous membranes with an active lesion through kissing, sexual contact, or routine skin-to-skin contact.

A nurse is speaking to a local women's group about the various types of cancer affecting the female reproductive tract. The nurse explains that ovarian cancer is the leading cause of death from gynecologic malignancies based on the understanding that this type of cancer: a. spreads more easily than other female reproductive cancers. b. arises from extremely rare types of cells that are resistant to treatment. c. typically manifests with vague symptoms resulting in late diagnosis. d. is closely associated with highly resistant sexually transmitted infections.

c. typically manifests with vague symptoms resulting in late diagnosis. Tumors of the ovary have been lethal largely because they present with nonspecific symptoms and therefore frequently are far advanced and inoperable by the time they are diagnosed. Ease of spread and types of cells involved are not reasons underlying the fatal nature of this type of cancer. Ovarian cancer is not associated with sexually transmitted infections. Cervical cancer is linked to human papillomavirus infection.

An elderly woman is seen in the clinic reporting a lesion on her labia majora and states that she has experienced some bleeding and itching as well. She states that this has been going on for approximately three months. She tells the nurse that she has not been to a health care provider in over 10 years. What diagnosis would the nurse expect the primary care provider to make? a. cervical cancer b. vaginal cancer c. vulvar cancer d. polyps

c. vulvar cancer The correct diagnosis for this client would be vulvar cancer due to the placement of the lesion, the itching, and the bleeding. The other cancers would have different symptoms and not apply to this client.

A woman of 16 weeks' gestation telephones the nurse because she has passed some "berry-like" blood clots and now has continued dark brown vaginal bleeding. Which action would the nurse instruct the woman to do? a. "Maintain bed rest, and count the number of perineal pads used." b. "Come to the health care facility if uterine contractions begin." c. "Continue normal activity, but take the pulse every hour." d. "Come to the health facility with any vaginal material passed."

d. "Come to the health facility with any vaginal material passed." This is a typical time in pregnancy for gestational trophoblastic disease to present. Asking the woman to bring any material passed vaginally would be important so it can be assessed for this.

The nurse is caring for a 52-year-old woman whose sisters and mother died of breast cancer. The client states, "My doctor wants me to take tamoxifen to help prevent breast cancer. What do you think?" What would be the nurse's best response? a. "We call this drug a chemotherapy agent." b. "Tamoxifen prevents osteoporosis." c. "I would recommend raloxifene." d. "Tamoxifen reduces the incidence."

d. "Tamoxifen reduces the incidence." Tamoxifen is referred to as a chemopreventive agent, not a chemotherapeutic agent. It does prevent osteoporosis, but this response does not address breast cancer prevention. Raloxifene is another drug that shows promise as a chemopreventive agent. Tamoxifen reduces breast cancer incidence by 49%.

A nurse is preparing a presentation on breast cancer for a local community group. When describing the age group estimated to have the highest risk, which group would the nurse cite? a. 30 to 39 years of age b. 40 to 49 years of age c. 50 to 59 years of age d. 60 to 69 years of age

d. 60 to 69 years of age The age group estimated to be at highest risk for breast cancer is ages 60 to 69 with an estimated risk of 1 out of 29. Those between ages 30 and 39 years have a 1 in 233 chance; those between ages 40 and 49 years have a 1 in 69 chance; and those between ages 50 and 59 have a 1 in 42 chance.

A pregnant woman with diabetes is having a glycosylated hemoglobin (HbA1C) level drawn. Which result would require the nurse to revise the client's plan of care? a. 5.5% b. 6.0% c. 7% d. 8.5%

d. 8.5% An HbA1C level of more than 8% indicates poor control and the need for intervention, necessitating a revision in the woman's plan of care.

A primigravida 28-year-old client is noted to have Rh negative blood and her husband is noted to be Rh positive. The nurse should prepare to administer RhoGAM after which diagnostic procedure? a. Contraction test b. Nonstress test c. Biophysical profile d. Amniocentesis

d. Amniocentesis Amniocentesis is a procedure requiring a needle to enter into the amniotic sac. There is a risk of mixing of the fetal and maternal blood which could result in blood incompatibility. A contraction test, a nonstress test, and biophysical profile are not invasive, so there would be no indication for Rho(D) immune globulin to be administered.

The LPN assists the RN while performing the Ortolani maneuver on a newborn. When asked by the mother the reason for this maneuver, which is the best response from the nurse? a. Spinal column movement b. Shoulder movement c. Clavicles for dislocation d. Hip for dislocation

d. Hip for dislocation Ortolani maneuver is used to assess the possibility of a dislocated hip in an infant. Ortolani maneuver does not assess for spinal column movement, shoulder movement, nor does it assess the clavicles for dislocation. There is no specific movement to assess for spinal column movement, shoulder movement, or clavicle dislocation.

What is the best rationale for trying to decrease the incidence of cold stress in the neonate? a. The neonate will stabilize its temperature by 8 hours after birth if kept warm and dry. b. Evaporative heat loss happens when the neonate is not bundled and does not have a hat on. c. It takes energy to keep warm, so the neonate has to remain in an extended position. d. If the neonate becomes cold stressed, it will eventually develop respiratory distress.

d. If the neonate becomes cold stressed, it will eventually develop respiratory distress. If cold stressed the infant eventually will develop respiratory distress; oxygen requirements rise, even before noting a change in temperature, glucose use increases, acids are released into the bloodstream and surfactant production decreases bringing on metabolic acidosis. A flexed position, not an extended position keeps the neonate warm.

A young couple are disappointed that they are not yet pregnant and are seeking assistance at the health clinic. After assessing their medical history, the nurse discovers the female has a history of several episodes of PID. The nurse predicts this may be a source of the infertility related to which factor? a. It causes anovulation due to interference with secretion of pituitary hormones. b. It causes changes in cervical mucus that make it less receptive to penetration by sperm. c. It causes sperm-agglutinating antibodies to be produced in the vagina. d. It interferes with the transport of ova due to tubal scarring.

d. It interferes with the transport of ova due to tubal scarring. Pelvic inflammatory disease results in scarring and adhesions of the tubes, leading to poor transport of ova. PID does not affect hormone metabolism, nor does it affect the production of cervical mucus. Antibodies are present only in a few cases and are unrelated to PID.

A nurse does an initial assessment on a newborn and notes a pulsation over the anterior fontanelle that corresponds with the newborn's heart rate. How would the nurse interpret this? a. This is an abnormal finding and needs to be reported immediately. b. If the fontanelle feels full, then this is normal. c. This finding is normal if the pulsation can also be palpated in the posterior fontanelle. d. It is normal to feel pulsations that correlate with the newborn's heart rate over the anterior fontanelle.

d. It is normal to feel pulsations that correlate with the newborn's heart rate over the anterior fontanelle. Feeling a pulsation over the fontanel correlating to the newborn's heart rate is normal. The pulsation should not be felt in the posterior fontanel. The fontanel should not be bulging under any circumstance in a newborn.

A nurse is caring for a 5-hour-old newborn. The primary care provider has asked the nurse to maintain the newborn's temperature between 97.7° F and 99.5° F (36.5° C and 37.5° C). Which nursing intervention would be the best approach to maintaining the temperature within the recommended range? a. Delay weighing the infant, as the scales may be cold. b. Use the stethoscope over the newborn's garment. c. Place the newborn's crib close to the outer wall in the room. d. Place the newborn skin-to-skin with the mother.

d. Place the newborn skin-to-skin with the mother. The nurse should place the newborn skin-to-skin with mother. This is the best way to help maintain the newborn's temperature as well as promoting breastfeeding and bonding between the mother and newborn. The nurse can weigh the infant as long as a warmed cover is placed on the scale. The stethoscope should be warmed before it makes contact with the infant's skin, rather than using the stethoscope over the garment because it may obscure the reading. The newborn's crib should not be placed close to the outer walls in the room to prevent heat loss through radiation.

The nursery nurse notes that one of the newborn infants has white patches on his tongue that look like milk curds. What action would be appropriate for the nurse to take? a.Wipe the tongue off vigorously to remove the white patches. b. Rinse the tongue off with sterile water and a cotton swab. c. Since it looks like a milk curd, no action is needed. d. Report the finding to the pediatrician.

d. Report the finding to the pediatrician. Although the finding looks like a milk curd, if the white patch remains after feeding, the pediatrician needs to be notified. The likely cause of the white patch on the tongue is a fungal infection called Candida albicans, which the newborn probably contracted while passing through the birth canal. The nurse should not try to remove the patches.

A client at 37 weeks' gestation presents to the emergency department with a BP 150/108 mm Hg, 1+ pedal edema, 1+ proteinuria, and normal deep tendon reflexes. Which assessment should the nurse prioritize as the client is administered magnesium sulfate IV? a. Urine protein b. Ability to sleep c. Hemoglobin d. Respiratory rate

d. Respiratory rate The level of magnesium in therapeutic range is 4 to 8 mg/dL. If magnesium toxicity occurs, one sign in the client will be a decrease in the respiratory rate and a potential respiratory arrest. Respiratory rate will be monitored when on this medication. The client's hemoglobin and ability to sleep are not factors for ongoing assessments for the client on magnesium sulfate. Urinary output is measured hourly on the preeclamptic client receiving magnesium sulfate, but urine protein is not an ongoing assessment.

A client with abnormal uterine bleeding is diagnosed with small ovarian cysts. The nurse has to educate the client on the importance of routine check-ups. Which assessment is most appropriate for this client's condition? a. Monitor gonadotropin level every month. b. Monitor blood sugar level every 15 days. c. Schedule periodic Papanicolau tests. d. Schedule an ultrasound every 3 to 6 months.

d. Schedule an ultrasound every 3 to 6 months. The nurse should monitor the client with ultrasound scans every 3 to 6 months. Monitoring gonadotropin level and blood sugar level and scheduling periodic Papanicolau tests are not important assessments for the client with small ovarian cysts.

What should the nurse expect for a full-term newborn's weight during the first few days of life? a. There is an increase in 3% to 5% of birth weight by day 3 in formula-fed babies. b. A formula-fed newborn should gain 3% to 5% of the initial birth weight in the first 48 hours, but a breastfed newborn may lose up to 3%. c. There is a loss of 5% to 10% of the birth weight in the first few days in breastfed infants only. d. There is a loss of 5% to 10% of birth weight in formula-fed and breastfed newborns.

d. There is a loss of 5% to 10% of birth weight in formula-fed and breastfed newborns. The nurse should expect the newborn who is breastfed or formula-fed to lose 5% to 10% of birth weight in the first few days of life.

A 36-year-old was diagnosed with uterine fibroids (leiomyomas). The nurse teaches the client to expect which clinical manifestation? a. decrease in fibroid size if pregnancy occurs b. diarrhea c. acute abdominal pain d. abnormal uterine bleeding

d. abnormal uterine bleeding Fibroids are dependent on estrogen and grow rapidly during the childbearing years unless menopause occurs. Abnormal uterine bleeding is a clinical manifestation. Diarrhea is not a factor; constipation and abdominal pain can occur if fibroids are getting larger.

A nurse is caring for a pregnant client with heart disease in a labor unit. Which intervention is most important in the first 48 hours postpartum? a. limiting sodium intake b. inspecting the extremities for edema c. ensuring that the client consumes a high fiber diet d. assessing for cardiac decompensation

d. assessing for cardiac decompensation The nurse should assess the client with heart disease for cardiac decompensation, which is most common from 28 to 32 weeks of gestation and in the first 48 hours postpartum. Limiting sodium intake, inspecting the extremities for edema, and ensuring that the client consumes a high-fiber diet are interventions during pregnancy not in the first 48 hours postpartum.

The nurse is required to assess a pregnant client who is reporting vaginal bleeding. Which nursing action is the priority? a. monitoring uterine contractility b. assessing signs of shock c. determining the amount of funneling d. assessing the amount and color of the bleeding

d. assessing the amount and color of the bleeding When the woman arrives and is admitted, assessing her vital signs, the amount and color of the bleeding, and current pain rating on a scale of 1 to 10 are the priorities. Assessing the signs of shock, monitoring uterine contractility, and determining the amount of funneling are not priority assessments when a pregnant woman complaining of vaginal bleeding is admitted to the hospital.

A client diagnosed with breast cancer is receiving chemotherapy. The nurse assesses the client for possible side effects of this therapy. Which side effect would the nurse report immediately to the health care provider? a. diarrhea b. nausea c. stomatitis d. bone marrow suppression

d. bone marrow suppression Typical side effects include nausea and vomiting, diarrhea or constipation, hair loss, weight loss, stomatitis, fatigue, and immunosuppression. The most serious is bone marrow suppression (myelosuppression). This causes an increased risk of infection, bleeding, and a reduced red blood cell count, which can lead to anemia.

Assessment of a client reveals evidence of a cystocele. The nurse interprets this as: a. herniation of the rectum into the vagina. b. protrusion of intestinal wall into the vagina. c. downward displacement of the cervix. d. bulging of the bladder into the vagina.

d. bulging of the bladder into the vagina. A cystocele is the bulging of the bladder into the vagina. A rectocele is a herniation of the rectum into the vagina. An enterocele is a protrusion of the intestinal wall into the vagina. A uterovaginal prolapse is the downward displacement of the cervix anywhere from low in the vagina to outside the vagina.

A nursing diagnosis of Risk for impaired tissue integrity would be most appropriate for which client? a. client with endometriosis b. client taking oral contraceptives c. client with a vaginal packing in place d. client having reconstructive breast surgery

d. client having reconstructive breast surgery Reconstructive breast surgery places the client at risk for insufficient blood supply to the muscle graft and skin, which can lead to tissue necrosis. Endometriosis or oral contraceptives are not generally associated with altered tissue perfusion. Pressure from vaginal packing can sometimes put pressure on the bladder neck and interfere with voiding.

A novice nurse asks to be assigned to the least complex antepartum client. Which condition would necessitate the least complex care requirements? a. preecalmpsia b. abruptio placenta c. placenta previa d. gestational hypertension

d. gestational hypertension Hypertensive disorders represent the most common complication of pregnancy. Gestational hypertension is elevated blood pressure without proteinuria, other signs of preeclampsia, or preexisting hypertension. Abruptio placenta (separation of the placenta from the uterine wall), placenta previa (placenta covering the cervical os), and preeclampsia are high-risk, potentially life-threatening conditions for the fetus and mother during labor and birth.

The nurse is reviewing the history and physical exam of a woman who has come to the clinic for a routine physical. Which factor would the nurse identify as increasing the client's risk for breast cancer? a. 39 years of age b. Asian race c. menarche at age 14 d. history of ovarian cancer

d. history of ovarian cancer A personal history of ovarian cancer is considered a risk factor for breast cancer. Typically, breast cancer is associated with aging (women over 50 years of age). Breast cancer is more common in Whtie women, but Black women are more likely to die of it. Early menarche (before 12 years of age) or late onset of menopause (after age 55 years) is associated with an increased risk for breast cancer.

The nurse is preparing a teaching plan for new parents about why newborns experience heat loss. Which information about newborns would the nurse include? a. thick skin with deep lying blood vessels b. enhanced shivering ability c. expanded stores of glucose and glycogen d. limited voluntary muscle activity

d. limited voluntary muscle activity Newborns have limited voluntary muscle activity or movement to produce heat. They have thin skin with blood vessels close to the surface. They cannot shiver to generate heat. They have limited stores of metabolic substances such as glucose and glycogen.

Which finding would the nurse most likely expect in a woman with endometrial polyps? a. abnormal vaginal bleeding b. vaginal discharge c. grayish-white mass d. metrorrhagia

d. metrorrhagia The most frequent clinical manifestation of women with endometrial polyps is metrorrhagia. Abnormal vaginal bleeding or discharge also may occur, but metrorrhagia is more common. Endocervical polyps are cherry red; cervical polyps are grayish-white.

The nurse explains to a pregnant client that she will need to take iron during her pregnancy after being diagnosed with iron-deficiency anemia. The nurse suggests that absorption of the supplemental iron can be increased by taking it with which substance? a. meals high in iron b. milk c. legumes d. orange juice

d. orange juice Anemia is a condition in which the blood is deficient in red blood cells, from an underlying cause. The woman needs to take iron to manufacture enough red blood cells. Taking an iron supplement will help improve her iron levels, and taking iron with foods containing ascorbic acid, such as orange juice, improves the absorption of iron.

A nurse is teaching newborn care to students. The nurse correctly identifies which mechanism as the predominant form of heat loss in the newborn? a. nonshivering thermogenesis b. lack of brown adipose tissue c. sweating and peripheral vasoconstriction d. radiation, convection, and conduction

d. radiation, convection, and conduction Heat loss in the newborn occurs primarily through radiation, convection, and conduction because of the newborn's large ratio of body surface to weight and because of the marked difference between core and skin temperatures. Nonshivering thermogenesis is a mechanism of heat production in the newborn. Lack of brown adipose tissue contributes to heat loss, particularly in premature infants, but it is not the predominant form of heat loss. Peripheral vasoconstriction is a method to increase heat production.

A woman is scheduled to undergo a modified radical mastectomy. Which information would the nurse include when describing this surgery to the client? a. the resulting concave appearance of the anterior chest b. sparing of the pectoral muscles and axillary lymph nodes c. wide excision of the tumor along with a 1-cm margin of normal tissue d. removal of breast tissue, axillary nodes, and some chest muscles

d. removal of breast tissue, axillary nodes, and some chest muscles A modified radical mastectomy involves removal of breast tissue, the axillary nodes, and some chest muscles but not the pectoralis major muscle. The surgery will not produce a concave anterior chest. With a simple mastectomy, all breast tissue, the nipple, and the areola are removed, but the axillary nodes and pectoral muscles are spared. A lumpectomy, or breast-conserving surgery, involves the wide local excision of the tumor along with a 1-cm margin of normal tissue.

Which factor would contribute to a high-risk pregnancy? a. blood type O positive b. first pregnancy at age 33 c. history of allergy to honey bee pollen d. type 1 diabetes

d. type 1 diabetes A woman with a history of diabetes has an increased risk for perinatal complications, including hypertension, preeclampsia, and neonatal hypoglycemia. The age of 33 without other risk factors does not increase risk, nor does type O-positive blood or environmental allergens.

A woman at 8 weeks' gestation is admitted for ectopic pregnancy. She is asking why this has occurred. The nurse knows that which factor is a known risk factor for ectopic pregnancy? a. high number of pregnancies b. multiple gestation pregnancy c. use of oral contraceptives d. use of IUD for contraception

d. use of IUD for contraception Use of an IUD with progesterone has a known increased risk for development of ectopic pregnancies. The nurse needs to complete a full history of the client to determine if she had any other risk factors for an ectopic pregnancy. Adhesions, scarring, and narrowing of the tubal lumen may block the zygote's progress to the uterus. Any condition or surgical procedure that can injure a fallopian tube increases the risk. Examples include salpingitis, infection of the fallopian tube, endometriosis, history of prior ectopic pregnancy, any type of tubal surgery, congenital malformation of the tube, and multiple elective abortions. Conditions that inhibit peristalsis of the tube can result in tubal pregnancy. Hormonal factors may play a role because tubal pregnancy occurs more frequently in women who take fertility drugs or who use progesterone intrauterine contraceptive devices (IUDs). A high number of pregnancies, multiple gestation pregnancy, and the use of oral contraceptives are not known risk factors for ectopic pregnancy.

At what point should the nurse expect a healthy newborn to pass meconium? a. before birth b. within 1 to 2 hours of birth c. by 12 to 18 hours of life d. within 24 hours after birth

d. within 24 hours after birth The healthy newborn should pass meconium within 24 hours of life.

The nurse is providing teaching to a new mother who is breastfeeding. The mother demonstrates understanding of teaching when she identifies which characteristics as being true of the stool of breastfed newborns? Select all that apply. a. formed in consistency b. completely odorless c. firm in shape d. yellowish gold color e. stringy to pasty consistency

d. yellowish gold color e. stringy to pasty consistency The stools of a breastfed newborn are yellowish gold in color. They are not firm in shape or solid. The smell is usually sour. A formula-fed infant's stools are formed in consistency, whereas a breastfed infant's stools are stringy to pasty in consistency.

All the options are signs of respiratory distress in the newborn except: a. grunting. b. nasal flaring. c. chest retractions. d. central cyanosis. e. respiratory rate >50 breaths/minute. f. coughing.

e. respiratory rate >50 breaths/minute. Coughing and sneezing are normal reflexes present in newborns. The expected respiratory rate of newborn is 30 to 60 breaths per minute.

The nursing student exhibits an understanding of pelvic organ prolapse (POP) when she correctly identifies which factors as contributing to its development? Select all that apply. vaginal birth obesity lifting chronic cough chronic diarrhea estrogen deficiency

vaginal birth obesity lifting chronic cough estrogen deficiency POP occurs as a result of weakness of the connective tissue and muscular support of pelvic organs due to the following factors: vaginal birth, obesity, lifting, chronic cough, straining upon defecation secondary to constipation, and estrogen deficiency.


Ensembles d'études connexes

Distributed computing interview questions

View Set

Chapter 12: Undernutrition Throughout the World

View Set

InQuizitive Sociology: Chapter 1

View Set

GEOG 363 - Homework & Exam Questions

View Set

Chapters 5-8 African American History (Book Self Test Answers)

View Set

states of matter - physical science

View Set